Use of point-of-care ultrasound (POCUS) for heart failure

Article Type
Changed
Tue, 08/17/2021 - 14:08

 

Case

A 65-year-old woman presents to the emergency department with a chief complaint of shortness of breath for 3 days. Medical history is notable for moderate chronic obstructive pulmonary disorder, systolic heart failure with last known ejection fraction (EF) of 35% and type 2 diabetes complicated by hyperglycemia when on steroids. You are talking the case over with colleagues and they suggest point-of-care ultrasound (POCUS) would be useful in her case.
 

Brief overview of the issue

Dr. Faye Farber
Dr. Faye Farber

Once mainly used by ED and critical care physicians, POCUS is now a tool that many hospitalists are using at the bedside. POCUS differs from traditional comprehensive ultrasounds in the following ways: POCUS is designed to answer a specific clinical question (as opposed to evaluating all organs in a specific region), POCUS exams are performed by the clinician who is formulating the clinical question (as opposed to by a consultative service such as cardiology and radiology), and POCUS can evaluate multiple organ systems (such as by evaluating a patient’s heart, lungs, and inferior vena cava to determine the etiology of hypoxia).

Hospitalist use of POCUS may include guiding procedures, aiding in diagnosis, and assessing effectiveness of treatment. Many high-quality studies have been published that support the use of POCUS and have proven that POCUS can decrease medical errors, help reach diagnoses in a more expedited fashion, and complement or replace more advanced imaging.

Dr. Yasmin Marcantonio of the Division of Hospital Medicine, Duke University Health System, Durham, NC
Dr. Yasmin Marcantonio

A challenge of POCUS is that it is user dependent and there are no established standards for hospitalists in POCUS training. As the Society of Hospital Medicine position statement on POCUS points out, there is a significant difference between skill levels required to obtain a certificate of completion for POCUS training and a certificate of competency in POCUS. Therefore, it is recommended hospitalists work with local credentialing committees to delineate the requirements for POCUS use.
 

Overview of the data

POCUS for initial assessment and diagnosis of heart failure (HF)

Use of POCUS in cases of suspected HF includes examination of the heart, lungs, and inferior vena cava (IVC). Cardiac ultrasound provides an estimated ejection fraction. Lung ultrasound (LUS) functions to examine for B lines and pleural effusions. The presence of more than three B lines per thoracic zone bilaterally suggests cardiogenic pulmonary edema. Scanning the IVC provides a noninvasive way to assess volume status and is especially helpful when body habitus prevents accurate assessment of jugular venous pressure.

Dr. Neil Stafford of the Division of Hospital Medicine, Duke University Health System, Durham, NC
Dr. Neil Stafford

Several studies have addressed the utility of bedside ultrasound in the initial assessment or diagnosis of acute decompensated heart failure (ADHF) in patients presenting with dyspnea in emergency or inpatient settings. Positive B lines are a useful finding, with high sensitivities, high specificities, and positive likelihood ratios. One large multicenter prospective study found LUS to have a sensitivity of 90.5%, specificity of 93.5%, and positive and negative LRs of 14.0 and 0.10, respectively.1 Another large multicenter prospective cohort study showed that LUS was more sensitive and more specific than chest x-ray (CXR) and brain natriuretic peptide in detecting ADHF.2 Additional POCUS findings that have shown relatively high sensitivities and specificities in the initial diagnosis of ADHF include pleural effusion, reduced left ventricular ejection fraction (LVEF), increased left ventricular end-diastolic dimension, and jugular venous distention.

Data also exists on assessments of ADHF using combinations of POCUS findings; for example, lung and cardiac ultrasound (LuCUS) protocols include an evaluation for B lines, assessment of IVC size and collapsibility, and determination of LVEF, although this has mainly been examined in ED patients. For patients who presented to the ED with undifferentiated dyspnea, one such study showed a specificity of 100% when a LuCUS protocol was used to diagnose ADHF while another study showed that the use of a LuCUS protocol changed management in 47% of patients.3,4 Of note, although each LuCUS protocol integrated the use of lung findings, IVC collapsibility, and LVEF, the exact protocols varied by institution. Finally, it has been established in multiple studies that LUS used in addition to standard workup including history and physical, labs, and electrocardiogram has been shown to increase diagnostic accuracy.2,5
 

 

 

Using POCUS to guide diuretic therapy in HF

Dr. Shree Menon of the Division of Hospital Medicine, Duke University Health System, Durham, NC
Dr. Shree Menon

To date, there have been multiple small studies published on the utility of daily POCUS in hospitalized patients with ADHF to help assess response to treatment and guide diuresis by looking for reduction in B lines on LUS or a change in IVC size or collapsibility. Volpicelli and colleagues showed that daily LUS was at least as good as daily CXR in monitoring response to therapy.6 Similarly, Mozzini and colleagues performed a randomized controlled trial of 120 patients admitted for ADHF who were randomized to a CXR group (who had a CXR performed on admission and discharge) and a LUS group (which was performed at admission, 24 hours, 48 hours, 72 hours, and discharge).7 This study found that the LUS group underwent a significantly higher number of diuretic dose adjustments as compared with the CXR group (P < .001) and had a modest improvement in LOS, compared with the CXR group. Specifically, median LOS was 8 days in CXR group (range, 4-17 days) and 7 days in the LUS group (range, 3-10 days; P < .001).

The impact of POCUS on length of stay (LOS) and readmissions

There is increasing data that POCUS can have meaningful impacts on patient-centered outcomes (morbidity, mortality, and readmission) while exposing patients to minimal discomfort, no venipuncture, and no radiation exposure. First, multiple studies looked at whether performing focused cardiac US of the IVC as a marker of volume status could predict readmission in patients hospitalized for ADHF.8,9 Both of these trials showed that plethoric, noncollapsible IVC at discharge were statistically significant predictors of readmission. In fact, Goonewardena and colleagues demonstrated that patients who required readmission had an enlarged IVC at discharge nearly 3 times more frequently (21% vs. 61%, P < .001) and abnormal IVC collapsibility 1.5 times more frequently (41% vs. 71%, P = .01) as compared with patients who remained out of the hospital.9

Similarly, a subsequent trial looked at whether IVC size on admission was of prognostic importance in patients hospitalized for ADHF and showed that admission IVC diameter was an independent predictor of both 90-day mortality (hazard ratio, 5.88; 95% confidence interval, 1.21-28.10; P = .025) and 90-day readmission (HR, 3.20; 95% CI, 1.24-8.21; P = .016).10 Additionally, LUS heart failure assessment for pulmonary congestion by counting B lines also showed that having more than 15 B lines prior to discharge was an independent predictor of readmission for ADHF at 6 months (HR, 11.74; 95% CI, 1.30-106.16).11

A challenge of POCUS: Obtaining competency

Dr. Adam Wachter
Dr. Adam Wachter

As previously noted, there are not yet any established standards for training and assessing hospitalists in POCUS. The SHM Position Statement on POCUS recommends the following criteria for training5: the training environment should be similar to the location in which the trainee will practice, training and feedback should occur in real time, the trainee should be taught specific applications of POCUS (such as cardiac US, LUS, and IVC US) as each application comes with unique skills and knowledge, clinical competence must be achieved and demonstrated, and continued education and feedback are necessary once competence is obtained.12 SHM recommends residency-based training pathways, training through a local or national program such as the SHM POCUS certificate program, or training through other medical societies for hospitalists already in practice.

 

 

Application of the data to our original case

Dr. Poonam Sharma of Duke University Health System, Durham, N.C.
Dr. Poonam Sharma

Targeted POCUS using the LuCUS protocol is performed and reveals three B lines in two lung zones bilaterally, moderate bilateral pleural effusions, EF 20%, and a noncollapsible IVC leading to a diagnosis of ADHF. Her ADHF is treated with intravenous diuresis. She is continued on her chronic maintenance chronic obstructive pulmonary disorder regimen but does not receive steroids, avoiding hyperglycemia that has complicated prior admissions. Over the next few days her respiratory and cardiac status is monitored using POCUS to assess her response to therapy and titrate her diuretics to her true dry weight, which was several pounds lower than her previously assumed dry weight. At discharge she is instructed to use the new dry weight which may avoid readmissions for HF.

Bottom line

POCUS improves diagnostic accuracy and facilitates volume assessment and management in acute decompensated heart failure.

Dr. Farber is a medical instructor at Duke University and hospitalist at Duke Regional Hospital, both in Durham, N.C. Dr. Marcantonio is a medical instructor in the department of internal medicine and department of pediatrics at Duke University and hospitalist at Duke University Hospital and Duke Regional Hospital. Dr. Stafford and Dr. Brooks are assistant professors of medicine and hospitalists at Duke Regional Hospital. Dr. Wachter is associate medical director at Duke Regional Hospital and assistant professor at Duke University. Dr. Menon is a hospitalist at Duke University. Dr. Sharma is associate medical director for clinical education at Duke Regional Hospital and associate professor of medicine at Duke University.

References

1. Pivetta E et al. Lung ultrasound integrated with clinical assessment for the diagnosis of acute decompensated heart failure in the emergency department: A randomized controlled trial. Eur J Heart Fail. 2019 Jun;21(6):754-66. doi: 10.1002/ejhf.1379.

2. Pivetta E et al. Lung ultrasound-implemented diagnosis of acute decompensated heart failure in the ED: A SIMEU multicenter study. Chest. 2015;148(1):202-10. doi: 10.1378/chest.14-2608.

3. Anderson KL et al. Diagnosing heart failure among acutely dyspneic patients with cardiac, inferior vena cava, and lung ultrasonography. Am J Emerg Med. 2013;31:1208-14. doi: 10.1016/j.ajem.2013.05.007.

4. Russell FM et al. Diagnosing acute heart failure in patients with undifferentiated dyspnea: A lung and cardiac ultrasound (LuCUS) protocol. Acad Emerg Med. 2015;22(2):182-91. doi:10.1111/acem.12570.

5. Maw AM et al. Diagnostic accuracy of point-of-care lung ultrasonography and chest radiography in adults with symptoms suggestive of acute decompensated heart failure: A systematic review and meta-analysis. JAMA Netw Open. 2019 Mar 1;2(3):e190703. doi:10.1001/jamanetworkopen.2019.0703.

6. Volpicelli G et al. Bedside ultrasound of the lung for the monitoring of acute decompensated heart failure. Am J Emerg Med. 2008 Jun;26(5):585-91. doi:10.1016/j.ajem.2007.09.014.

7. Mozzini C et al. Lung ultrasound in internal medicine efficiently drives the management of patients with heart failure and speeds up the discharge time. Intern Emerg Med. 2018 Jan;13(1):27-33. doi: 10.1007/s11739-017-1738-1.

8. Laffin LJ et al. Focused cardiac ultrasound as a predictor of readmission in acute decompensated heart failure. Int J Cardiovasc Imaging. 2018;34(7):1075-9. doi:10.1007/s10554-018-1317-1.

9. Goonewardena SN et al. Comparison of hand-carried ultrasound assessment of the inferior vena cava and N-terminal pro-brain natriuretic peptide for predicting readmission after hospitalization for acute decompensated heart failure. JACC Cardiovasc Imaging. 2008;1(5):595-601. doi:10.1016/j.jcmg.2008.06.005.

10. Cubo-Romano P et al. Admission inferior vena cava measurements are associated with mortality after hospitalization for acute decompensated heart failure. J Hosp Med. 2016 Nov;11(11):778-84. doi: 10.1002/jhm.2620.

11. Gargani L et al. Persistent pulmonary congestion before discharge predicts rehospitalization in heart failure: A lung ultrasound study. Cardiovasc Ultrasound. 2015 Sep 4;13:40. doi: 10.1186/s12947-015-0033-4.

12. Soni NJ et al. Point-of-care ultrasound for hospitalists: A Position Statement of the Society of Hospital Medicine. J Hosp Med. 2019 Jan 2;14:E1-6. doi: 10.12788/jhm.3079.

Key points

  • Studies have found POCUS improves the diagnosis of acute decompensated heart failure in patients presenting with dyspnea.
  • Daily evaluation with POCUS has decreased length of stay in acute decompensated heart failure.
  • Credentialing requirements for hospitalists to use POCUS for clinical care vary by hospital.

Additional reading

Maw AM and Soni NJ. Annals for hospitalists inpatient notes – why should hospitalists use point-of-care ultrasound? Ann Intern Med. 2018 Apr 17;168(8):HO2-HO3. doi: 10.7326/M18-0367.

Lewiss RE. “The ultrasound looked fine”: Point of care ultrasound and patient safety. AHRQ’s Patient Safety Network. WebM&M: Case Studies. 2018 Jul 1. https://psnet.ahrq.gov/web-mm/ultrasound-looked-fine-point-care-ultrasound-and-patient-safety.

Quiz: Testing your POCUS knowledge

POCUS is increasingly prevalent in hospital medicine, but use varies among different disease processes. Which organ system ultrasound or lab test would be most helpful in the following scenario?

An acutely dyspneic patient with no past medical history presents to the ED. Chest x-ray is equivocal. Of the following, which study best confirms a diagnosis of acute decompensated heart failure?

A. Brain natriuretic peptide

B. Point-of-care cardiac ultrasound

C. Point-of-care lung ultrasound

D. Point-of-care inferior vena cava ultrasound

Answer

C. Point-of-care lung ultrasound

Multiple studies, including three systematic reviews, have shown that point-of-care lung ultrasound has high sensitivity and specificity to evaluate for B lines as a marker for cardiogenic pulmonary edema. Point-of-care ultrasound of ejection fraction and inferior vena cava have not been evaluated by systematic review although one randomized, controlled trial showed that an EF less than 45% had 74% specificity and 77% sensitivity and IVC collapsibility index less than 20% had an 86% specificity and 52% sensitivity for detection of acute decompensated heart failure. This same study showed that the combination of cardiac, lung, and IVC point-of-care ultrasound had 100% specificity for diagnosing acute decompensated heart failure. In the future, health care providers could rely on this multiorgan evaluation with point-of-care ultrasound to confirm a diagnosis of acute decompensated heart failure in a dyspneic patient.

Publications
Topics
Sections

 

Case

A 65-year-old woman presents to the emergency department with a chief complaint of shortness of breath for 3 days. Medical history is notable for moderate chronic obstructive pulmonary disorder, systolic heart failure with last known ejection fraction (EF) of 35% and type 2 diabetes complicated by hyperglycemia when on steroids. You are talking the case over with colleagues and they suggest point-of-care ultrasound (POCUS) would be useful in her case.
 

Brief overview of the issue

Dr. Faye Farber
Dr. Faye Farber

Once mainly used by ED and critical care physicians, POCUS is now a tool that many hospitalists are using at the bedside. POCUS differs from traditional comprehensive ultrasounds in the following ways: POCUS is designed to answer a specific clinical question (as opposed to evaluating all organs in a specific region), POCUS exams are performed by the clinician who is formulating the clinical question (as opposed to by a consultative service such as cardiology and radiology), and POCUS can evaluate multiple organ systems (such as by evaluating a patient’s heart, lungs, and inferior vena cava to determine the etiology of hypoxia).

Hospitalist use of POCUS may include guiding procedures, aiding in diagnosis, and assessing effectiveness of treatment. Many high-quality studies have been published that support the use of POCUS and have proven that POCUS can decrease medical errors, help reach diagnoses in a more expedited fashion, and complement or replace more advanced imaging.

Dr. Yasmin Marcantonio of the Division of Hospital Medicine, Duke University Health System, Durham, NC
Dr. Yasmin Marcantonio

A challenge of POCUS is that it is user dependent and there are no established standards for hospitalists in POCUS training. As the Society of Hospital Medicine position statement on POCUS points out, there is a significant difference between skill levels required to obtain a certificate of completion for POCUS training and a certificate of competency in POCUS. Therefore, it is recommended hospitalists work with local credentialing committees to delineate the requirements for POCUS use.
 

Overview of the data

POCUS for initial assessment and diagnosis of heart failure (HF)

Use of POCUS in cases of suspected HF includes examination of the heart, lungs, and inferior vena cava (IVC). Cardiac ultrasound provides an estimated ejection fraction. Lung ultrasound (LUS) functions to examine for B lines and pleural effusions. The presence of more than three B lines per thoracic zone bilaterally suggests cardiogenic pulmonary edema. Scanning the IVC provides a noninvasive way to assess volume status and is especially helpful when body habitus prevents accurate assessment of jugular venous pressure.

Dr. Neil Stafford of the Division of Hospital Medicine, Duke University Health System, Durham, NC
Dr. Neil Stafford

Several studies have addressed the utility of bedside ultrasound in the initial assessment or diagnosis of acute decompensated heart failure (ADHF) in patients presenting with dyspnea in emergency or inpatient settings. Positive B lines are a useful finding, with high sensitivities, high specificities, and positive likelihood ratios. One large multicenter prospective study found LUS to have a sensitivity of 90.5%, specificity of 93.5%, and positive and negative LRs of 14.0 and 0.10, respectively.1 Another large multicenter prospective cohort study showed that LUS was more sensitive and more specific than chest x-ray (CXR) and brain natriuretic peptide in detecting ADHF.2 Additional POCUS findings that have shown relatively high sensitivities and specificities in the initial diagnosis of ADHF include pleural effusion, reduced left ventricular ejection fraction (LVEF), increased left ventricular end-diastolic dimension, and jugular venous distention.

Data also exists on assessments of ADHF using combinations of POCUS findings; for example, lung and cardiac ultrasound (LuCUS) protocols include an evaluation for B lines, assessment of IVC size and collapsibility, and determination of LVEF, although this has mainly been examined in ED patients. For patients who presented to the ED with undifferentiated dyspnea, one such study showed a specificity of 100% when a LuCUS protocol was used to diagnose ADHF while another study showed that the use of a LuCUS protocol changed management in 47% of patients.3,4 Of note, although each LuCUS protocol integrated the use of lung findings, IVC collapsibility, and LVEF, the exact protocols varied by institution. Finally, it has been established in multiple studies that LUS used in addition to standard workup including history and physical, labs, and electrocardiogram has been shown to increase diagnostic accuracy.2,5
 

 

 

Using POCUS to guide diuretic therapy in HF

Dr. Shree Menon of the Division of Hospital Medicine, Duke University Health System, Durham, NC
Dr. Shree Menon

To date, there have been multiple small studies published on the utility of daily POCUS in hospitalized patients with ADHF to help assess response to treatment and guide diuresis by looking for reduction in B lines on LUS or a change in IVC size or collapsibility. Volpicelli and colleagues showed that daily LUS was at least as good as daily CXR in monitoring response to therapy.6 Similarly, Mozzini and colleagues performed a randomized controlled trial of 120 patients admitted for ADHF who were randomized to a CXR group (who had a CXR performed on admission and discharge) and a LUS group (which was performed at admission, 24 hours, 48 hours, 72 hours, and discharge).7 This study found that the LUS group underwent a significantly higher number of diuretic dose adjustments as compared with the CXR group (P < .001) and had a modest improvement in LOS, compared with the CXR group. Specifically, median LOS was 8 days in CXR group (range, 4-17 days) and 7 days in the LUS group (range, 3-10 days; P < .001).

The impact of POCUS on length of stay (LOS) and readmissions

There is increasing data that POCUS can have meaningful impacts on patient-centered outcomes (morbidity, mortality, and readmission) while exposing patients to minimal discomfort, no venipuncture, and no radiation exposure. First, multiple studies looked at whether performing focused cardiac US of the IVC as a marker of volume status could predict readmission in patients hospitalized for ADHF.8,9 Both of these trials showed that plethoric, noncollapsible IVC at discharge were statistically significant predictors of readmission. In fact, Goonewardena and colleagues demonstrated that patients who required readmission had an enlarged IVC at discharge nearly 3 times more frequently (21% vs. 61%, P < .001) and abnormal IVC collapsibility 1.5 times more frequently (41% vs. 71%, P = .01) as compared with patients who remained out of the hospital.9

Similarly, a subsequent trial looked at whether IVC size on admission was of prognostic importance in patients hospitalized for ADHF and showed that admission IVC diameter was an independent predictor of both 90-day mortality (hazard ratio, 5.88; 95% confidence interval, 1.21-28.10; P = .025) and 90-day readmission (HR, 3.20; 95% CI, 1.24-8.21; P = .016).10 Additionally, LUS heart failure assessment for pulmonary congestion by counting B lines also showed that having more than 15 B lines prior to discharge was an independent predictor of readmission for ADHF at 6 months (HR, 11.74; 95% CI, 1.30-106.16).11

A challenge of POCUS: Obtaining competency

Dr. Adam Wachter
Dr. Adam Wachter

As previously noted, there are not yet any established standards for training and assessing hospitalists in POCUS. The SHM Position Statement on POCUS recommends the following criteria for training5: the training environment should be similar to the location in which the trainee will practice, training and feedback should occur in real time, the trainee should be taught specific applications of POCUS (such as cardiac US, LUS, and IVC US) as each application comes with unique skills and knowledge, clinical competence must be achieved and demonstrated, and continued education and feedback are necessary once competence is obtained.12 SHM recommends residency-based training pathways, training through a local or national program such as the SHM POCUS certificate program, or training through other medical societies for hospitalists already in practice.

 

 

Application of the data to our original case

Dr. Poonam Sharma of Duke University Health System, Durham, N.C.
Dr. Poonam Sharma

Targeted POCUS using the LuCUS protocol is performed and reveals three B lines in two lung zones bilaterally, moderate bilateral pleural effusions, EF 20%, and a noncollapsible IVC leading to a diagnosis of ADHF. Her ADHF is treated with intravenous diuresis. She is continued on her chronic maintenance chronic obstructive pulmonary disorder regimen but does not receive steroids, avoiding hyperglycemia that has complicated prior admissions. Over the next few days her respiratory and cardiac status is monitored using POCUS to assess her response to therapy and titrate her diuretics to her true dry weight, which was several pounds lower than her previously assumed dry weight. At discharge she is instructed to use the new dry weight which may avoid readmissions for HF.

Bottom line

POCUS improves diagnostic accuracy and facilitates volume assessment and management in acute decompensated heart failure.

Dr. Farber is a medical instructor at Duke University and hospitalist at Duke Regional Hospital, both in Durham, N.C. Dr. Marcantonio is a medical instructor in the department of internal medicine and department of pediatrics at Duke University and hospitalist at Duke University Hospital and Duke Regional Hospital. Dr. Stafford and Dr. Brooks are assistant professors of medicine and hospitalists at Duke Regional Hospital. Dr. Wachter is associate medical director at Duke Regional Hospital and assistant professor at Duke University. Dr. Menon is a hospitalist at Duke University. Dr. Sharma is associate medical director for clinical education at Duke Regional Hospital and associate professor of medicine at Duke University.

References

1. Pivetta E et al. Lung ultrasound integrated with clinical assessment for the diagnosis of acute decompensated heart failure in the emergency department: A randomized controlled trial. Eur J Heart Fail. 2019 Jun;21(6):754-66. doi: 10.1002/ejhf.1379.

2. Pivetta E et al. Lung ultrasound-implemented diagnosis of acute decompensated heart failure in the ED: A SIMEU multicenter study. Chest. 2015;148(1):202-10. doi: 10.1378/chest.14-2608.

3. Anderson KL et al. Diagnosing heart failure among acutely dyspneic patients with cardiac, inferior vena cava, and lung ultrasonography. Am J Emerg Med. 2013;31:1208-14. doi: 10.1016/j.ajem.2013.05.007.

4. Russell FM et al. Diagnosing acute heart failure in patients with undifferentiated dyspnea: A lung and cardiac ultrasound (LuCUS) protocol. Acad Emerg Med. 2015;22(2):182-91. doi:10.1111/acem.12570.

5. Maw AM et al. Diagnostic accuracy of point-of-care lung ultrasonography and chest radiography in adults with symptoms suggestive of acute decompensated heart failure: A systematic review and meta-analysis. JAMA Netw Open. 2019 Mar 1;2(3):e190703. doi:10.1001/jamanetworkopen.2019.0703.

6. Volpicelli G et al. Bedside ultrasound of the lung for the monitoring of acute decompensated heart failure. Am J Emerg Med. 2008 Jun;26(5):585-91. doi:10.1016/j.ajem.2007.09.014.

7. Mozzini C et al. Lung ultrasound in internal medicine efficiently drives the management of patients with heart failure and speeds up the discharge time. Intern Emerg Med. 2018 Jan;13(1):27-33. doi: 10.1007/s11739-017-1738-1.

8. Laffin LJ et al. Focused cardiac ultrasound as a predictor of readmission in acute decompensated heart failure. Int J Cardiovasc Imaging. 2018;34(7):1075-9. doi:10.1007/s10554-018-1317-1.

9. Goonewardena SN et al. Comparison of hand-carried ultrasound assessment of the inferior vena cava and N-terminal pro-brain natriuretic peptide for predicting readmission after hospitalization for acute decompensated heart failure. JACC Cardiovasc Imaging. 2008;1(5):595-601. doi:10.1016/j.jcmg.2008.06.005.

10. Cubo-Romano P et al. Admission inferior vena cava measurements are associated with mortality after hospitalization for acute decompensated heart failure. J Hosp Med. 2016 Nov;11(11):778-84. doi: 10.1002/jhm.2620.

11. Gargani L et al. Persistent pulmonary congestion before discharge predicts rehospitalization in heart failure: A lung ultrasound study. Cardiovasc Ultrasound. 2015 Sep 4;13:40. doi: 10.1186/s12947-015-0033-4.

12. Soni NJ et al. Point-of-care ultrasound for hospitalists: A Position Statement of the Society of Hospital Medicine. J Hosp Med. 2019 Jan 2;14:E1-6. doi: 10.12788/jhm.3079.

Key points

  • Studies have found POCUS improves the diagnosis of acute decompensated heart failure in patients presenting with dyspnea.
  • Daily evaluation with POCUS has decreased length of stay in acute decompensated heart failure.
  • Credentialing requirements for hospitalists to use POCUS for clinical care vary by hospital.

Additional reading

Maw AM and Soni NJ. Annals for hospitalists inpatient notes – why should hospitalists use point-of-care ultrasound? Ann Intern Med. 2018 Apr 17;168(8):HO2-HO3. doi: 10.7326/M18-0367.

Lewiss RE. “The ultrasound looked fine”: Point of care ultrasound and patient safety. AHRQ’s Patient Safety Network. WebM&M: Case Studies. 2018 Jul 1. https://psnet.ahrq.gov/web-mm/ultrasound-looked-fine-point-care-ultrasound-and-patient-safety.

Quiz: Testing your POCUS knowledge

POCUS is increasingly prevalent in hospital medicine, but use varies among different disease processes. Which organ system ultrasound or lab test would be most helpful in the following scenario?

An acutely dyspneic patient with no past medical history presents to the ED. Chest x-ray is equivocal. Of the following, which study best confirms a diagnosis of acute decompensated heart failure?

A. Brain natriuretic peptide

B. Point-of-care cardiac ultrasound

C. Point-of-care lung ultrasound

D. Point-of-care inferior vena cava ultrasound

Answer

C. Point-of-care lung ultrasound

Multiple studies, including three systematic reviews, have shown that point-of-care lung ultrasound has high sensitivity and specificity to evaluate for B lines as a marker for cardiogenic pulmonary edema. Point-of-care ultrasound of ejection fraction and inferior vena cava have not been evaluated by systematic review although one randomized, controlled trial showed that an EF less than 45% had 74% specificity and 77% sensitivity and IVC collapsibility index less than 20% had an 86% specificity and 52% sensitivity for detection of acute decompensated heart failure. This same study showed that the combination of cardiac, lung, and IVC point-of-care ultrasound had 100% specificity for diagnosing acute decompensated heart failure. In the future, health care providers could rely on this multiorgan evaluation with point-of-care ultrasound to confirm a diagnosis of acute decompensated heart failure in a dyspneic patient.

 

Case

A 65-year-old woman presents to the emergency department with a chief complaint of shortness of breath for 3 days. Medical history is notable for moderate chronic obstructive pulmonary disorder, systolic heart failure with last known ejection fraction (EF) of 35% and type 2 diabetes complicated by hyperglycemia when on steroids. You are talking the case over with colleagues and they suggest point-of-care ultrasound (POCUS) would be useful in her case.
 

Brief overview of the issue

Dr. Faye Farber
Dr. Faye Farber

Once mainly used by ED and critical care physicians, POCUS is now a tool that many hospitalists are using at the bedside. POCUS differs from traditional comprehensive ultrasounds in the following ways: POCUS is designed to answer a specific clinical question (as opposed to evaluating all organs in a specific region), POCUS exams are performed by the clinician who is formulating the clinical question (as opposed to by a consultative service such as cardiology and radiology), and POCUS can evaluate multiple organ systems (such as by evaluating a patient’s heart, lungs, and inferior vena cava to determine the etiology of hypoxia).

Hospitalist use of POCUS may include guiding procedures, aiding in diagnosis, and assessing effectiveness of treatment. Many high-quality studies have been published that support the use of POCUS and have proven that POCUS can decrease medical errors, help reach diagnoses in a more expedited fashion, and complement or replace more advanced imaging.

Dr. Yasmin Marcantonio of the Division of Hospital Medicine, Duke University Health System, Durham, NC
Dr. Yasmin Marcantonio

A challenge of POCUS is that it is user dependent and there are no established standards for hospitalists in POCUS training. As the Society of Hospital Medicine position statement on POCUS points out, there is a significant difference between skill levels required to obtain a certificate of completion for POCUS training and a certificate of competency in POCUS. Therefore, it is recommended hospitalists work with local credentialing committees to delineate the requirements for POCUS use.
 

Overview of the data

POCUS for initial assessment and diagnosis of heart failure (HF)

Use of POCUS in cases of suspected HF includes examination of the heart, lungs, and inferior vena cava (IVC). Cardiac ultrasound provides an estimated ejection fraction. Lung ultrasound (LUS) functions to examine for B lines and pleural effusions. The presence of more than three B lines per thoracic zone bilaterally suggests cardiogenic pulmonary edema. Scanning the IVC provides a noninvasive way to assess volume status and is especially helpful when body habitus prevents accurate assessment of jugular venous pressure.

Dr. Neil Stafford of the Division of Hospital Medicine, Duke University Health System, Durham, NC
Dr. Neil Stafford

Several studies have addressed the utility of bedside ultrasound in the initial assessment or diagnosis of acute decompensated heart failure (ADHF) in patients presenting with dyspnea in emergency or inpatient settings. Positive B lines are a useful finding, with high sensitivities, high specificities, and positive likelihood ratios. One large multicenter prospective study found LUS to have a sensitivity of 90.5%, specificity of 93.5%, and positive and negative LRs of 14.0 and 0.10, respectively.1 Another large multicenter prospective cohort study showed that LUS was more sensitive and more specific than chest x-ray (CXR) and brain natriuretic peptide in detecting ADHF.2 Additional POCUS findings that have shown relatively high sensitivities and specificities in the initial diagnosis of ADHF include pleural effusion, reduced left ventricular ejection fraction (LVEF), increased left ventricular end-diastolic dimension, and jugular venous distention.

Data also exists on assessments of ADHF using combinations of POCUS findings; for example, lung and cardiac ultrasound (LuCUS) protocols include an evaluation for B lines, assessment of IVC size and collapsibility, and determination of LVEF, although this has mainly been examined in ED patients. For patients who presented to the ED with undifferentiated dyspnea, one such study showed a specificity of 100% when a LuCUS protocol was used to diagnose ADHF while another study showed that the use of a LuCUS protocol changed management in 47% of patients.3,4 Of note, although each LuCUS protocol integrated the use of lung findings, IVC collapsibility, and LVEF, the exact protocols varied by institution. Finally, it has been established in multiple studies that LUS used in addition to standard workup including history and physical, labs, and electrocardiogram has been shown to increase diagnostic accuracy.2,5
 

 

 

Using POCUS to guide diuretic therapy in HF

Dr. Shree Menon of the Division of Hospital Medicine, Duke University Health System, Durham, NC
Dr. Shree Menon

To date, there have been multiple small studies published on the utility of daily POCUS in hospitalized patients with ADHF to help assess response to treatment and guide diuresis by looking for reduction in B lines on LUS or a change in IVC size or collapsibility. Volpicelli and colleagues showed that daily LUS was at least as good as daily CXR in monitoring response to therapy.6 Similarly, Mozzini and colleagues performed a randomized controlled trial of 120 patients admitted for ADHF who were randomized to a CXR group (who had a CXR performed on admission and discharge) and a LUS group (which was performed at admission, 24 hours, 48 hours, 72 hours, and discharge).7 This study found that the LUS group underwent a significantly higher number of diuretic dose adjustments as compared with the CXR group (P < .001) and had a modest improvement in LOS, compared with the CXR group. Specifically, median LOS was 8 days in CXR group (range, 4-17 days) and 7 days in the LUS group (range, 3-10 days; P < .001).

The impact of POCUS on length of stay (LOS) and readmissions

There is increasing data that POCUS can have meaningful impacts on patient-centered outcomes (morbidity, mortality, and readmission) while exposing patients to minimal discomfort, no venipuncture, and no radiation exposure. First, multiple studies looked at whether performing focused cardiac US of the IVC as a marker of volume status could predict readmission in patients hospitalized for ADHF.8,9 Both of these trials showed that plethoric, noncollapsible IVC at discharge were statistically significant predictors of readmission. In fact, Goonewardena and colleagues demonstrated that patients who required readmission had an enlarged IVC at discharge nearly 3 times more frequently (21% vs. 61%, P < .001) and abnormal IVC collapsibility 1.5 times more frequently (41% vs. 71%, P = .01) as compared with patients who remained out of the hospital.9

Similarly, a subsequent trial looked at whether IVC size on admission was of prognostic importance in patients hospitalized for ADHF and showed that admission IVC diameter was an independent predictor of both 90-day mortality (hazard ratio, 5.88; 95% confidence interval, 1.21-28.10; P = .025) and 90-day readmission (HR, 3.20; 95% CI, 1.24-8.21; P = .016).10 Additionally, LUS heart failure assessment for pulmonary congestion by counting B lines also showed that having more than 15 B lines prior to discharge was an independent predictor of readmission for ADHF at 6 months (HR, 11.74; 95% CI, 1.30-106.16).11

A challenge of POCUS: Obtaining competency

Dr. Adam Wachter
Dr. Adam Wachter

As previously noted, there are not yet any established standards for training and assessing hospitalists in POCUS. The SHM Position Statement on POCUS recommends the following criteria for training5: the training environment should be similar to the location in which the trainee will practice, training and feedback should occur in real time, the trainee should be taught specific applications of POCUS (such as cardiac US, LUS, and IVC US) as each application comes with unique skills and knowledge, clinical competence must be achieved and demonstrated, and continued education and feedback are necessary once competence is obtained.12 SHM recommends residency-based training pathways, training through a local or national program such as the SHM POCUS certificate program, or training through other medical societies for hospitalists already in practice.

 

 

Application of the data to our original case

Dr. Poonam Sharma of Duke University Health System, Durham, N.C.
Dr. Poonam Sharma

Targeted POCUS using the LuCUS protocol is performed and reveals three B lines in two lung zones bilaterally, moderate bilateral pleural effusions, EF 20%, and a noncollapsible IVC leading to a diagnosis of ADHF. Her ADHF is treated with intravenous diuresis. She is continued on her chronic maintenance chronic obstructive pulmonary disorder regimen but does not receive steroids, avoiding hyperglycemia that has complicated prior admissions. Over the next few days her respiratory and cardiac status is monitored using POCUS to assess her response to therapy and titrate her diuretics to her true dry weight, which was several pounds lower than her previously assumed dry weight. At discharge she is instructed to use the new dry weight which may avoid readmissions for HF.

Bottom line

POCUS improves diagnostic accuracy and facilitates volume assessment and management in acute decompensated heart failure.

Dr. Farber is a medical instructor at Duke University and hospitalist at Duke Regional Hospital, both in Durham, N.C. Dr. Marcantonio is a medical instructor in the department of internal medicine and department of pediatrics at Duke University and hospitalist at Duke University Hospital and Duke Regional Hospital. Dr. Stafford and Dr. Brooks are assistant professors of medicine and hospitalists at Duke Regional Hospital. Dr. Wachter is associate medical director at Duke Regional Hospital and assistant professor at Duke University. Dr. Menon is a hospitalist at Duke University. Dr. Sharma is associate medical director for clinical education at Duke Regional Hospital and associate professor of medicine at Duke University.

References

1. Pivetta E et al. Lung ultrasound integrated with clinical assessment for the diagnosis of acute decompensated heart failure in the emergency department: A randomized controlled trial. Eur J Heart Fail. 2019 Jun;21(6):754-66. doi: 10.1002/ejhf.1379.

2. Pivetta E et al. Lung ultrasound-implemented diagnosis of acute decompensated heart failure in the ED: A SIMEU multicenter study. Chest. 2015;148(1):202-10. doi: 10.1378/chest.14-2608.

3. Anderson KL et al. Diagnosing heart failure among acutely dyspneic patients with cardiac, inferior vena cava, and lung ultrasonography. Am J Emerg Med. 2013;31:1208-14. doi: 10.1016/j.ajem.2013.05.007.

4. Russell FM et al. Diagnosing acute heart failure in patients with undifferentiated dyspnea: A lung and cardiac ultrasound (LuCUS) protocol. Acad Emerg Med. 2015;22(2):182-91. doi:10.1111/acem.12570.

5. Maw AM et al. Diagnostic accuracy of point-of-care lung ultrasonography and chest radiography in adults with symptoms suggestive of acute decompensated heart failure: A systematic review and meta-analysis. JAMA Netw Open. 2019 Mar 1;2(3):e190703. doi:10.1001/jamanetworkopen.2019.0703.

6. Volpicelli G et al. Bedside ultrasound of the lung for the monitoring of acute decompensated heart failure. Am J Emerg Med. 2008 Jun;26(5):585-91. doi:10.1016/j.ajem.2007.09.014.

7. Mozzini C et al. Lung ultrasound in internal medicine efficiently drives the management of patients with heart failure and speeds up the discharge time. Intern Emerg Med. 2018 Jan;13(1):27-33. doi: 10.1007/s11739-017-1738-1.

8. Laffin LJ et al. Focused cardiac ultrasound as a predictor of readmission in acute decompensated heart failure. Int J Cardiovasc Imaging. 2018;34(7):1075-9. doi:10.1007/s10554-018-1317-1.

9. Goonewardena SN et al. Comparison of hand-carried ultrasound assessment of the inferior vena cava and N-terminal pro-brain natriuretic peptide for predicting readmission after hospitalization for acute decompensated heart failure. JACC Cardiovasc Imaging. 2008;1(5):595-601. doi:10.1016/j.jcmg.2008.06.005.

10. Cubo-Romano P et al. Admission inferior vena cava measurements are associated with mortality after hospitalization for acute decompensated heart failure. J Hosp Med. 2016 Nov;11(11):778-84. doi: 10.1002/jhm.2620.

11. Gargani L et al. Persistent pulmonary congestion before discharge predicts rehospitalization in heart failure: A lung ultrasound study. Cardiovasc Ultrasound. 2015 Sep 4;13:40. doi: 10.1186/s12947-015-0033-4.

12. Soni NJ et al. Point-of-care ultrasound for hospitalists: A Position Statement of the Society of Hospital Medicine. J Hosp Med. 2019 Jan 2;14:E1-6. doi: 10.12788/jhm.3079.

Key points

  • Studies have found POCUS improves the diagnosis of acute decompensated heart failure in patients presenting with dyspnea.
  • Daily evaluation with POCUS has decreased length of stay in acute decompensated heart failure.
  • Credentialing requirements for hospitalists to use POCUS for clinical care vary by hospital.

Additional reading

Maw AM and Soni NJ. Annals for hospitalists inpatient notes – why should hospitalists use point-of-care ultrasound? Ann Intern Med. 2018 Apr 17;168(8):HO2-HO3. doi: 10.7326/M18-0367.

Lewiss RE. “The ultrasound looked fine”: Point of care ultrasound and patient safety. AHRQ’s Patient Safety Network. WebM&M: Case Studies. 2018 Jul 1. https://psnet.ahrq.gov/web-mm/ultrasound-looked-fine-point-care-ultrasound-and-patient-safety.

Quiz: Testing your POCUS knowledge

POCUS is increasingly prevalent in hospital medicine, but use varies among different disease processes. Which organ system ultrasound or lab test would be most helpful in the following scenario?

An acutely dyspneic patient with no past medical history presents to the ED. Chest x-ray is equivocal. Of the following, which study best confirms a diagnosis of acute decompensated heart failure?

A. Brain natriuretic peptide

B. Point-of-care cardiac ultrasound

C. Point-of-care lung ultrasound

D. Point-of-care inferior vena cava ultrasound

Answer

C. Point-of-care lung ultrasound

Multiple studies, including three systematic reviews, have shown that point-of-care lung ultrasound has high sensitivity and specificity to evaluate for B lines as a marker for cardiogenic pulmonary edema. Point-of-care ultrasound of ejection fraction and inferior vena cava have not been evaluated by systematic review although one randomized, controlled trial showed that an EF less than 45% had 74% specificity and 77% sensitivity and IVC collapsibility index less than 20% had an 86% specificity and 52% sensitivity for detection of acute decompensated heart failure. This same study showed that the combination of cardiac, lung, and IVC point-of-care ultrasound had 100% specificity for diagnosing acute decompensated heart failure. In the future, health care providers could rely on this multiorgan evaluation with point-of-care ultrasound to confirm a diagnosis of acute decompensated heart failure in a dyspneic patient.

Publications
Publications
Topics
Article Type
Sections
Disallow All Ads
Content Gating
No Gating (article Unlocked/Free)
Alternative CME
Disqus Comments
Default
Use ProPublica
Hide sidebar & use full width
render the right sidebar.
Conference Recap Checkbox
Not Conference Recap
Clinical Edge
Display the Slideshow in this Article
Medscape Article
Display survey writer
Reuters content
Disable Inline Native ads
WebMD Article

Catheter ablation of AFib improves quality of life more than medications do

Article Type
Changed
Mon, 06/08/2020 - 13:03

Background: Catheter ablation of AFib (primarily pulmonary vein isolation) has been shown to result in better maintenance of sinus rhythm than medications. Small studies of QOL have shown mixed results. Larger trials were needed.

Dr. Neil Stafford of the Division of Hospital Medicine, Duke University Health System, Durham, NC
Dr. Neil Stafford


Study design: Open-label randomized multisite clinical trial of catheter ablation (pulmonary vein isolation with additional ablation procedure at the treating physician discretion) versus standard rate and/or rhythm control medications (chosen by clinician discretion). Patients were included for paroxysmal or persistent AFib and either age 65 years or older or age younger than 65 years with one additional stroke risk factor. Quality of life surveys – the Atrial Fibrillation Effect on Quality of Life (AFEQT) questionnaire and the Mayo AF-Specific Symptom Inventory (MAFSI) – were completed at baseline, and at 3, 12, 24, 36, 48, and 60 months.

Setting: 126 centers in 10 countries.

Synopsis: The study included 2,204 patients with median age of 68 years, diagnosed with AFib a median of 1.1 years prior, who were followed for a median of 48 months. The median CHA2DS2-VASc score was 3.0.

Self-reported AFib dropped from 86.0% to 21.1% in the ablation group and from 83.7% to 39.8% in the medication group at 12 months. The AFEQT score (range 0-100, higher score indicating better QOL) increased from 62.9 to 86.4 in the ablation group and increased from 63.1 to 80.9 in the medication group (for a mean difference of 5.3 points [95% confidence interval, 3.7-6.9; P less than .001] favoring ablation). MAFSI symptom frequency score and symptom severity score also showed improvement in symptoms favoring ablation. Post hoc subgroup analysis showed that those with the most severe symptoms had the largest benefit from ablation.

The primary limitation is the lack of patient blinding (may bias self-reported symptoms).

While the CABANA trial efficacy study (published separately) showed that catheter ablation results in no significant difference in the combined outcome of death, disabling stroke, serious bleeding, or cardiac arrest, the CABANA QOL study, reviewed here, shows that ablation does result in improved QOL and reduced symptoms, compared with medical therapy.

Bottom line: Catheter ablation of AFib can be done safely and successfully at experienced centers. In patients with AFib-related symptoms, ablation reduces symptoms and improves QOL somewhat more than medications do. The most severely symptomatic patients appear to obtain the most benefit.

Citation: Packer DL et al. Effect of catheter ablation vs. antiarrhythmic drug therapy on mortality, stroke, bleeding, and cardiac arrest among patients with atrial fibrillation: The CABANA Randomized Clinical Trial. JAMA. 2019 Mar 15. doi: 10.1001/jama.2019.0693.

Dr. Stafford is a hospitalist at Duke University Health System.

Publications
Topics
Sections

Background: Catheter ablation of AFib (primarily pulmonary vein isolation) has been shown to result in better maintenance of sinus rhythm than medications. Small studies of QOL have shown mixed results. Larger trials were needed.

Dr. Neil Stafford of the Division of Hospital Medicine, Duke University Health System, Durham, NC
Dr. Neil Stafford


Study design: Open-label randomized multisite clinical trial of catheter ablation (pulmonary vein isolation with additional ablation procedure at the treating physician discretion) versus standard rate and/or rhythm control medications (chosen by clinician discretion). Patients were included for paroxysmal or persistent AFib and either age 65 years or older or age younger than 65 years with one additional stroke risk factor. Quality of life surveys – the Atrial Fibrillation Effect on Quality of Life (AFEQT) questionnaire and the Mayo AF-Specific Symptom Inventory (MAFSI) – were completed at baseline, and at 3, 12, 24, 36, 48, and 60 months.

Setting: 126 centers in 10 countries.

Synopsis: The study included 2,204 patients with median age of 68 years, diagnosed with AFib a median of 1.1 years prior, who were followed for a median of 48 months. The median CHA2DS2-VASc score was 3.0.

Self-reported AFib dropped from 86.0% to 21.1% in the ablation group and from 83.7% to 39.8% in the medication group at 12 months. The AFEQT score (range 0-100, higher score indicating better QOL) increased from 62.9 to 86.4 in the ablation group and increased from 63.1 to 80.9 in the medication group (for a mean difference of 5.3 points [95% confidence interval, 3.7-6.9; P less than .001] favoring ablation). MAFSI symptom frequency score and symptom severity score also showed improvement in symptoms favoring ablation. Post hoc subgroup analysis showed that those with the most severe symptoms had the largest benefit from ablation.

The primary limitation is the lack of patient blinding (may bias self-reported symptoms).

While the CABANA trial efficacy study (published separately) showed that catheter ablation results in no significant difference in the combined outcome of death, disabling stroke, serious bleeding, or cardiac arrest, the CABANA QOL study, reviewed here, shows that ablation does result in improved QOL and reduced symptoms, compared with medical therapy.

Bottom line: Catheter ablation of AFib can be done safely and successfully at experienced centers. In patients with AFib-related symptoms, ablation reduces symptoms and improves QOL somewhat more than medications do. The most severely symptomatic patients appear to obtain the most benefit.

Citation: Packer DL et al. Effect of catheter ablation vs. antiarrhythmic drug therapy on mortality, stroke, bleeding, and cardiac arrest among patients with atrial fibrillation: The CABANA Randomized Clinical Trial. JAMA. 2019 Mar 15. doi: 10.1001/jama.2019.0693.

Dr. Stafford is a hospitalist at Duke University Health System.

Background: Catheter ablation of AFib (primarily pulmonary vein isolation) has been shown to result in better maintenance of sinus rhythm than medications. Small studies of QOL have shown mixed results. Larger trials were needed.

Dr. Neil Stafford of the Division of Hospital Medicine, Duke University Health System, Durham, NC
Dr. Neil Stafford


Study design: Open-label randomized multisite clinical trial of catheter ablation (pulmonary vein isolation with additional ablation procedure at the treating physician discretion) versus standard rate and/or rhythm control medications (chosen by clinician discretion). Patients were included for paroxysmal or persistent AFib and either age 65 years or older or age younger than 65 years with one additional stroke risk factor. Quality of life surveys – the Atrial Fibrillation Effect on Quality of Life (AFEQT) questionnaire and the Mayo AF-Specific Symptom Inventory (MAFSI) – were completed at baseline, and at 3, 12, 24, 36, 48, and 60 months.

Setting: 126 centers in 10 countries.

Synopsis: The study included 2,204 patients with median age of 68 years, diagnosed with AFib a median of 1.1 years prior, who were followed for a median of 48 months. The median CHA2DS2-VASc score was 3.0.

Self-reported AFib dropped from 86.0% to 21.1% in the ablation group and from 83.7% to 39.8% in the medication group at 12 months. The AFEQT score (range 0-100, higher score indicating better QOL) increased from 62.9 to 86.4 in the ablation group and increased from 63.1 to 80.9 in the medication group (for a mean difference of 5.3 points [95% confidence interval, 3.7-6.9; P less than .001] favoring ablation). MAFSI symptom frequency score and symptom severity score also showed improvement in symptoms favoring ablation. Post hoc subgroup analysis showed that those with the most severe symptoms had the largest benefit from ablation.

The primary limitation is the lack of patient blinding (may bias self-reported symptoms).

While the CABANA trial efficacy study (published separately) showed that catheter ablation results in no significant difference in the combined outcome of death, disabling stroke, serious bleeding, or cardiac arrest, the CABANA QOL study, reviewed here, shows that ablation does result in improved QOL and reduced symptoms, compared with medical therapy.

Bottom line: Catheter ablation of AFib can be done safely and successfully at experienced centers. In patients with AFib-related symptoms, ablation reduces symptoms and improves QOL somewhat more than medications do. The most severely symptomatic patients appear to obtain the most benefit.

Citation: Packer DL et al. Effect of catheter ablation vs. antiarrhythmic drug therapy on mortality, stroke, bleeding, and cardiac arrest among patients with atrial fibrillation: The CABANA Randomized Clinical Trial. JAMA. 2019 Mar 15. doi: 10.1001/jama.2019.0693.

Dr. Stafford is a hospitalist at Duke University Health System.

Publications
Publications
Topics
Article Type
Sections
Disallow All Ads
Content Gating
No Gating (article Unlocked/Free)
Alternative CME
Disqus Comments
Default
Use ProPublica
Hide sidebar & use full width
render the right sidebar.
Conference Recap Checkbox
Not Conference Recap

Anticoagulant therapy for AFib in patients with end-stage renal disease

Article Type
Changed
Fri, 08/23/2019 - 15:53

Warfarin or apixaban are sensible options

 

Case

A 78-year-old woman with end-stage renal disease (ESRD) is hospitalized with cellulitis and is incidentally found to be in atrial fibrillation. She does not have a history of mitral stenosis, nor does she have a prosthetic valve. She does have a history of hypertension, diabetes, and prior stroke without residual deficits.

Dr. Faye Farber
Dr. Faye Farber

After counseling her about the risk of stroke associated with atrial fibrillation (AFib) she makes it clear she is interested in pharmacologic therapy to minimize her risk of stroke and asks what medication you would recommend for anticoagulation.

Brief overview of the issue

Anticoagulation for AFib is indicated for stroke prophylaxis in patients with an elevated risk of stroke. The CHA2DS2-VASc score is useful in calculating an individual patient’s risk of stroke and as a decision tool to determine who would benefit from anticoagulation, and it is recommended in the American Heart Association guidelines.1

Low-risk patients (CHA2DS2-VASc score of 0 in men or 1 in women) should not be started on anticoagulation for stroke prophylaxis. For anyone with a risk factor, other than being female, anticoagulation is indicated and should be considered.

Dr. Neil Stafford of the Division of Hospital Medicine, Duke University Health System, Durham, NC
Dr. Neil Stafford

The guideline recommends anticoagulant therapy, not antiplatelet agents. For most of the recent past, this has meant a vitamin K antagonist (warfarin) or sometimes a low-molecular-weight heparin injected subcutaneously. Over the past decade, however, with the approval of multiple direct oral anticoagulants (DOACs), nonwarfarin oral anticoagulation has grown in popularity as the prophylactic medication of choice.2

While the data for patients with preserved renal function is robust, there is far less data to guide decision making for patients with end-stage renal disease.

Overview of the data

Until the introduction of DOACs, warfarin was the main agent used for stroke prophylaxis in patients with end-stage kidney disease and AFib. Professional guidelines favored warfarin for these patients who were mostly excluded from DOAC trials. Specialized conferences also looked at this issue.

Dr. Suchita Sata

The Kidney Disease: Improving Global Outcomes (KDIGO) Controversies Conference, which reviewed chronic kidney disease and arrhythmias, noted that there were no randomized controlled trials that examined the efficacy and safety of anticoagulation in chronic kidney disease patients with estimated creatinine clearance less than 30 mL/min. They remarked that there was insufficient high-quality evidence to recommend warfarin for the prevention of stroke in patients with AFib and dialysis-dependent chronic kidney disease.

Since, according to other trials, DOACs had better safety profiles in other populations, the conference noted that lower-dose apixaban (2.5 mg orally twice daily) or rivaroxaban (15 mg daily) may be considered in this population until clinical safety data were available. Furthermore, the conference recommended that these patients be treated with a multidisciplinary approach in regards to anticoagulation and have an annual reevaluation of treatment goals, along with a risk-benefit assessment.3

Dr. Rami Abdo of Division of Hospital Medicine, Duke University Health System, Durham, NC
Dr. Rami Abdo

Since the publication of the 2018 AHA guidelines and the guidance document that resulted from the KDIGO conference, additional research has been published comparing anticoagulation with a DOAC versus warfarin for AFib in patients with ESRD.

“Outcomes associated with apixaban use in patients with end-stage kidney disease and atrial fibrillation in the United States” was an observational, retrospective, cohort study that compared outcomes in dialysis patients who took warfarin for AFib with those who took apixaban.4 Patients’ data was taken from the U.S. Renal Data System database and were included in the final analysis if they had ESRD, a recent diagnosis of AFib or atrial flutter, and a new prescription for either warfarin or apixaban. Outcome measures were stroke or systemic embolism, major bleeding (critical site, transfusion, or death), gastrointestinal bleeding, intracranial bleeding, or death. Drug usage and compliance were assessed using Medicare Part D prescription information.

Dr. Megan Brooks, assistant professor of medicine, Duke University, Durham, N.C.
Dr. Megan Brooks

A total of 25,523 patients met the inclusion/exclusion criteria and had taken either warfarin (n = 23,172) or apixaban (n = 2,351). To account for selection bias in these cohorts, a subset of the warfarin patients was selected based on prognostic score matching. The prognostic score was calculated from the baseline characteristics (which included age, stroke history, diabetes, smoking, antiplatelet medication, liver disease, prior bleeding, and CHA2DS2-VASc score). Kaplan-Meier and Cox regression analysis were used to give hazard ratios and 95% confidence intervals for each outcome measure. Prespecified subgroup analyses were conducted to compare apixaban doses, where 44% were prescribed 5 mg b.i.d. and 56% were prescribed 2.5 mg b.i.d..

In the study, patients in the apixaban group had a significantly lower risk of major bleeding as compared with the warfarin group (HR, 0.72; 95% CI, 0.59-0.87; P less than .001) with overall high rates of major bleeding in both groups at 19.7 and 22.9 per 100 patient-years in the apixaban group and warfarin group, respectively. There was no difference in the rate of stroke/systemic embolism between patients receiving apixaban and warfarin (HR, 0.88; 95% CI, 0.69-1.12; P = .29). There was a nonsignificant trend toward decreased risk of GI bleeding in the apixaban group and no significant differences between the groups in the rates of intracranial bleeding. Apixaban was also associated with a nonsignificant trend toward lower risk of mortality (HR, 0.85; 95% CI, 0.71-1.01; P = .06).

Dr. Shree Menon of the Division of Hospital Medicine, Duke University Health System, Durham, NC
Dr. Shree Menon

Notably, censoring rates because of expired prescriptions or a 1-month gap between prescriptions were high in both groups and the majority of censoring occurred within the first 12 months. Additionally, in dose specific analyses, patients receiving the 5-mg, twice-daily dose were found to have statistically significant decreases in risk of stroke/systemic embolism (P = .035) and mortality (P = .005) as compared with the 2.5-mg, twice-daily dose without significant differences in GI or intracranial bleeding.

There are three ongoing, open-label, randomized, controlled trials examining anticoagulation for nonvalvular AFib in patients with ESRD on hemodialysis with two comparing apixaban to warfarin (or derivative) and the other warfarin versus no anticoagulation.5 All trials are in adult patients with documented AFib and CHA2DS2-VASc score of at least 2. AKADIA (Germany based) plans to enroll 222 patients and compares a vitamin K antagonist (INR goal, 2-3) with 2.5-mg b.i.d. apixaban patients with ESRD on hemodialysis for at least 3 months with primary outcome of major and clinically relevant nonmajor bleeding and secondary outcome of thromboembolic events, as well as apixaban levels pre- and post hemodialysis.

Dr. Adam Wachter
Dr. Adam Wachter

RENAL-AF (U.S. based) plans to enrolled 762 patients and compares 5-mg b.i.d. apixaban (with 2.5 mg for selected patients) with warfarin in people of chronic hemodialysis with primary outcome of days to first major or clinically relevant nonmajor bleeding event and secondary outcome of stroke, systemic embolism, mortality, adherence and plasma apixaban levels. AVKDIAL (France based) plans to enroll 855 patients and compares no anticoagulation with vitamin K antagonists in patients on hemodialysis for at least 1 month, with primary outcome of cumulative incidence of severe bleeding and thrombosis.

 

 

Application of the data to our original case

Our patient is Medicare age with ESRD and newly diagnosed nonvalvular AFib. Recent data suggests apixaban could be used for stroke prevention instead of the prior standard of care, warfarin. This approach is supported in the 2019 guidelines.1

Dr. Poonam Sharma of Duke University Health System, Durham, N.C.
Dr. Poonam Sharma

Patients with ESRD have an increased risk of bleeding and apixaban was shown to have less bleeding complications than warfarin in this analysis. However, only standard-dose apixaban was associated with a statistically significant lower risk of stroke/systemic embolism, major bleeding, and death. Reduced-dose apixaban had a lower risk of major bleeding but no difference for stroke/systemic embolism or death. Reduced-dose apixaban is used for patients who have two out of the following three criteria: aged at least 80 years, weight of at least 60 kg, and creatinine of at least 1.5 mg/dL. Therefore, many Medicare-age patients with ESRD would not be indicated for the dose of apixaban that was shown to improve the most important outcomes of stroke/SE and death.

It may still be beneficial to use apixaban in this patient since it appears to work as well as warfarin for stroke/systemic embolism prevention with less bleeding complications.

Bottom line

For patients who have decided to pursue an anticoagulation strategy for stroke prevention in AFib and have end-stage renal disease, either warfarin or apixaban are sensible options.

Dr. Farber is a medical instructor at Duke University Health System in Durham, N.C. Dr. Stafford is a medical instructor at Duke University. Dr. Sata is assistant professor of medicine at Duke University. Dr. Abdo and Dr. Menon are hospitalists at Duke University. Dr. Brooks is assistant professor of medicine at Duke University. Dr. Wachter is associate medical director at Duke Regional Hospital and assistant professor of medicine at Duke University. Dr. Sharma is associate medical director for clinical education in hospital medicine at Duke Regional Hospital and assistant professor of medicine at Duke University.

References

1. January CT et al. 2019 AHA/ACC/HRS focused update of the 2014 AHA/ACC/HRS guideline for the management of patients with atrial fibrillation: A report of the American College of Cardiology/American Heart Association Task Force on Clinical Practice Guidelines and the Heart Rhythm Society. Circulation. 2019;139. doi: 1161/CIR.0000000000000665.

2. Lippi G et al. Direct oral anticoagulants: Analysis of worldwide use and popularity using Google Trends. Ann Transl Med. 2017 Aug; 5(16):322. doi: 10.21037/atm.2017.06.65.

3. Turakhia MP et al. Chronic kidney disease and arrhythmias: Conclusions from a Kidney Disease: Improving Global Outcomes (KDIGO) Controversies Conference. Eur Heart J. 2018 Jun 21;39(24):2314-25. doi: 10.1093/eurheartj/ehy060.

4. Siontis KC et al. Outcomes associated with apixaban use in patients with end-stage kidney disease and atrial fibrillation in the United States. Circulation. 2018 Oct 9;138(15):1519-29. doi: 10.1161/CIRCULATIONAHA.118.035418.

5. Nigwekar SU et al. Long-term anticoagulation for patient receiving dialysis: Tilting the benefit-to-risk ratio? Circulation. 2018 Oct 9;138(15):1530-3. doi: 10.1161/CIRCULATIONAHA.118.037091.

Key points

  • According to 2019 American Heart Association guidelines, warfarin or apixaban are reasonable options for stroke prevention for patients who have end-stage renal disease and who plan for anticoagulation because of atrial fibrillation.
  • Recent observational data suggests that apixaban may be safer than warfarin in this population.
  • Several randomized, controlled trials are ongoing that may help determine the optimal agent to use in this setting.
  • Until more definitive data is available, a reasonable approach is to discuss the risks and benefits of various treatment strategies with patients, and engage a multidisciplinary team (cardiologist, nephrologist, primary care provider, pharmacist) in the decision making process.
 

 

Additional reading

January CT et al. 2019 AHA/ACC/HRS focused update of the 2014 AHA/ACC/HRS guideline for the management of patients with atrial fibrillation: A report of the American College of Cardiology/American Heart Association Task Force on Clinical Practice Guidelines and the Heart Rhythm Society. Circulation. 2019;139. doi: 1161/CIR.0000000000000665.

Nigwekar SU et al. Long-term anticoagulation for patient receiving dialysis: Tilting the benefit to risk ratio? Circulation. 2018 Oct 9;138(15):1530-3. doi: 10.1161/CIRCULATIONAHA.118.037091.

Garlo KG et al. Demystifying the benefits and harms of anticoagulation for atrial fibrillation in chronic kidney disease. Clin J Am Soc Nephrol 2019;14:125-36. doi: 10.2215/CJN.06430518.

Quiz

Two days ago you admitted a 72-year-old woman with end-stage renal disease on dialysis who had developed new-onset atrial fibrillation causing a mild acute diastolic congestive heart failure exacerbation. Transthoracic ECG showed a preserved left ventricular ejection fraction and no significant valvular disease. After two sessions of dialysis in the hospital and initiation of a beta-blocker for control of her heart rate, she is stable and ready for discharge. Her discharge weight is 75 kg.

Which of the following recommendations should you make to this patient regarding anticoagulation for prevention of stroke and systemic embolism from atrial fibrillation?

A. Take warfarin with a international normalized ratio goal of 2.5.

B. Take apixaban 2.5 mg twice a day.

C. Take apixaban 5 mg twice a day.

D. Discuss the risks/benefits of various treatment approaches with the patient, and involve the hospital pharmacist as well as the patient’s nephrologist, cardiologist, and/or primary care provider in the decision making process to reach a consensus and to ensure a safe follow-up plan.

The best answer is D. While A, B, and C are all reasonable approaches based on the available data and current guidelines, the best approach is to involve the patient and the multidisciplinary team in the decision making process. When more clinical trial data becomes available in the future, the optimal approach to managing patients such as this one may become clearer, but until then it makes sense to take into account individual patient characteristics and patient preferences.

Publications
Topics
Sections

Warfarin or apixaban are sensible options

Warfarin or apixaban are sensible options

 

Case

A 78-year-old woman with end-stage renal disease (ESRD) is hospitalized with cellulitis and is incidentally found to be in atrial fibrillation. She does not have a history of mitral stenosis, nor does she have a prosthetic valve. She does have a history of hypertension, diabetes, and prior stroke without residual deficits.

Dr. Faye Farber
Dr. Faye Farber

After counseling her about the risk of stroke associated with atrial fibrillation (AFib) she makes it clear she is interested in pharmacologic therapy to minimize her risk of stroke and asks what medication you would recommend for anticoagulation.

Brief overview of the issue

Anticoagulation for AFib is indicated for stroke prophylaxis in patients with an elevated risk of stroke. The CHA2DS2-VASc score is useful in calculating an individual patient’s risk of stroke and as a decision tool to determine who would benefit from anticoagulation, and it is recommended in the American Heart Association guidelines.1

Low-risk patients (CHA2DS2-VASc score of 0 in men or 1 in women) should not be started on anticoagulation for stroke prophylaxis. For anyone with a risk factor, other than being female, anticoagulation is indicated and should be considered.

Dr. Neil Stafford of the Division of Hospital Medicine, Duke University Health System, Durham, NC
Dr. Neil Stafford

The guideline recommends anticoagulant therapy, not antiplatelet agents. For most of the recent past, this has meant a vitamin K antagonist (warfarin) or sometimes a low-molecular-weight heparin injected subcutaneously. Over the past decade, however, with the approval of multiple direct oral anticoagulants (DOACs), nonwarfarin oral anticoagulation has grown in popularity as the prophylactic medication of choice.2

While the data for patients with preserved renal function is robust, there is far less data to guide decision making for patients with end-stage renal disease.

Overview of the data

Until the introduction of DOACs, warfarin was the main agent used for stroke prophylaxis in patients with end-stage kidney disease and AFib. Professional guidelines favored warfarin for these patients who were mostly excluded from DOAC trials. Specialized conferences also looked at this issue.

Dr. Suchita Sata

The Kidney Disease: Improving Global Outcomes (KDIGO) Controversies Conference, which reviewed chronic kidney disease and arrhythmias, noted that there were no randomized controlled trials that examined the efficacy and safety of anticoagulation in chronic kidney disease patients with estimated creatinine clearance less than 30 mL/min. They remarked that there was insufficient high-quality evidence to recommend warfarin for the prevention of stroke in patients with AFib and dialysis-dependent chronic kidney disease.

Since, according to other trials, DOACs had better safety profiles in other populations, the conference noted that lower-dose apixaban (2.5 mg orally twice daily) or rivaroxaban (15 mg daily) may be considered in this population until clinical safety data were available. Furthermore, the conference recommended that these patients be treated with a multidisciplinary approach in regards to anticoagulation and have an annual reevaluation of treatment goals, along with a risk-benefit assessment.3

Dr. Rami Abdo of Division of Hospital Medicine, Duke University Health System, Durham, NC
Dr. Rami Abdo

Since the publication of the 2018 AHA guidelines and the guidance document that resulted from the KDIGO conference, additional research has been published comparing anticoagulation with a DOAC versus warfarin for AFib in patients with ESRD.

“Outcomes associated with apixaban use in patients with end-stage kidney disease and atrial fibrillation in the United States” was an observational, retrospective, cohort study that compared outcomes in dialysis patients who took warfarin for AFib with those who took apixaban.4 Patients’ data was taken from the U.S. Renal Data System database and were included in the final analysis if they had ESRD, a recent diagnosis of AFib or atrial flutter, and a new prescription for either warfarin or apixaban. Outcome measures were stroke or systemic embolism, major bleeding (critical site, transfusion, or death), gastrointestinal bleeding, intracranial bleeding, or death. Drug usage and compliance were assessed using Medicare Part D prescription information.

Dr. Megan Brooks, assistant professor of medicine, Duke University, Durham, N.C.
Dr. Megan Brooks

A total of 25,523 patients met the inclusion/exclusion criteria and had taken either warfarin (n = 23,172) or apixaban (n = 2,351). To account for selection bias in these cohorts, a subset of the warfarin patients was selected based on prognostic score matching. The prognostic score was calculated from the baseline characteristics (which included age, stroke history, diabetes, smoking, antiplatelet medication, liver disease, prior bleeding, and CHA2DS2-VASc score). Kaplan-Meier and Cox regression analysis were used to give hazard ratios and 95% confidence intervals for each outcome measure. Prespecified subgroup analyses were conducted to compare apixaban doses, where 44% were prescribed 5 mg b.i.d. and 56% were prescribed 2.5 mg b.i.d..

In the study, patients in the apixaban group had a significantly lower risk of major bleeding as compared with the warfarin group (HR, 0.72; 95% CI, 0.59-0.87; P less than .001) with overall high rates of major bleeding in both groups at 19.7 and 22.9 per 100 patient-years in the apixaban group and warfarin group, respectively. There was no difference in the rate of stroke/systemic embolism between patients receiving apixaban and warfarin (HR, 0.88; 95% CI, 0.69-1.12; P = .29). There was a nonsignificant trend toward decreased risk of GI bleeding in the apixaban group and no significant differences between the groups in the rates of intracranial bleeding. Apixaban was also associated with a nonsignificant trend toward lower risk of mortality (HR, 0.85; 95% CI, 0.71-1.01; P = .06).

Dr. Shree Menon of the Division of Hospital Medicine, Duke University Health System, Durham, NC
Dr. Shree Menon

Notably, censoring rates because of expired prescriptions or a 1-month gap between prescriptions were high in both groups and the majority of censoring occurred within the first 12 months. Additionally, in dose specific analyses, patients receiving the 5-mg, twice-daily dose were found to have statistically significant decreases in risk of stroke/systemic embolism (P = .035) and mortality (P = .005) as compared with the 2.5-mg, twice-daily dose without significant differences in GI or intracranial bleeding.

There are three ongoing, open-label, randomized, controlled trials examining anticoagulation for nonvalvular AFib in patients with ESRD on hemodialysis with two comparing apixaban to warfarin (or derivative) and the other warfarin versus no anticoagulation.5 All trials are in adult patients with documented AFib and CHA2DS2-VASc score of at least 2. AKADIA (Germany based) plans to enroll 222 patients and compares a vitamin K antagonist (INR goal, 2-3) with 2.5-mg b.i.d. apixaban patients with ESRD on hemodialysis for at least 3 months with primary outcome of major and clinically relevant nonmajor bleeding and secondary outcome of thromboembolic events, as well as apixaban levels pre- and post hemodialysis.

Dr. Adam Wachter
Dr. Adam Wachter

RENAL-AF (U.S. based) plans to enrolled 762 patients and compares 5-mg b.i.d. apixaban (with 2.5 mg for selected patients) with warfarin in people of chronic hemodialysis with primary outcome of days to first major or clinically relevant nonmajor bleeding event and secondary outcome of stroke, systemic embolism, mortality, adherence and plasma apixaban levels. AVKDIAL (France based) plans to enroll 855 patients and compares no anticoagulation with vitamin K antagonists in patients on hemodialysis for at least 1 month, with primary outcome of cumulative incidence of severe bleeding and thrombosis.

 

 

Application of the data to our original case

Our patient is Medicare age with ESRD and newly diagnosed nonvalvular AFib. Recent data suggests apixaban could be used for stroke prevention instead of the prior standard of care, warfarin. This approach is supported in the 2019 guidelines.1

Dr. Poonam Sharma of Duke University Health System, Durham, N.C.
Dr. Poonam Sharma

Patients with ESRD have an increased risk of bleeding and apixaban was shown to have less bleeding complications than warfarin in this analysis. However, only standard-dose apixaban was associated with a statistically significant lower risk of stroke/systemic embolism, major bleeding, and death. Reduced-dose apixaban had a lower risk of major bleeding but no difference for stroke/systemic embolism or death. Reduced-dose apixaban is used for patients who have two out of the following three criteria: aged at least 80 years, weight of at least 60 kg, and creatinine of at least 1.5 mg/dL. Therefore, many Medicare-age patients with ESRD would not be indicated for the dose of apixaban that was shown to improve the most important outcomes of stroke/SE and death.

It may still be beneficial to use apixaban in this patient since it appears to work as well as warfarin for stroke/systemic embolism prevention with less bleeding complications.

Bottom line

For patients who have decided to pursue an anticoagulation strategy for stroke prevention in AFib and have end-stage renal disease, either warfarin or apixaban are sensible options.

Dr. Farber is a medical instructor at Duke University Health System in Durham, N.C. Dr. Stafford is a medical instructor at Duke University. Dr. Sata is assistant professor of medicine at Duke University. Dr. Abdo and Dr. Menon are hospitalists at Duke University. Dr. Brooks is assistant professor of medicine at Duke University. Dr. Wachter is associate medical director at Duke Regional Hospital and assistant professor of medicine at Duke University. Dr. Sharma is associate medical director for clinical education in hospital medicine at Duke Regional Hospital and assistant professor of medicine at Duke University.

References

1. January CT et al. 2019 AHA/ACC/HRS focused update of the 2014 AHA/ACC/HRS guideline for the management of patients with atrial fibrillation: A report of the American College of Cardiology/American Heart Association Task Force on Clinical Practice Guidelines and the Heart Rhythm Society. Circulation. 2019;139. doi: 1161/CIR.0000000000000665.

2. Lippi G et al. Direct oral anticoagulants: Analysis of worldwide use and popularity using Google Trends. Ann Transl Med. 2017 Aug; 5(16):322. doi: 10.21037/atm.2017.06.65.

3. Turakhia MP et al. Chronic kidney disease and arrhythmias: Conclusions from a Kidney Disease: Improving Global Outcomes (KDIGO) Controversies Conference. Eur Heart J. 2018 Jun 21;39(24):2314-25. doi: 10.1093/eurheartj/ehy060.

4. Siontis KC et al. Outcomes associated with apixaban use in patients with end-stage kidney disease and atrial fibrillation in the United States. Circulation. 2018 Oct 9;138(15):1519-29. doi: 10.1161/CIRCULATIONAHA.118.035418.

5. Nigwekar SU et al. Long-term anticoagulation for patient receiving dialysis: Tilting the benefit-to-risk ratio? Circulation. 2018 Oct 9;138(15):1530-3. doi: 10.1161/CIRCULATIONAHA.118.037091.

Key points

  • According to 2019 American Heart Association guidelines, warfarin or apixaban are reasonable options for stroke prevention for patients who have end-stage renal disease and who plan for anticoagulation because of atrial fibrillation.
  • Recent observational data suggests that apixaban may be safer than warfarin in this population.
  • Several randomized, controlled trials are ongoing that may help determine the optimal agent to use in this setting.
  • Until more definitive data is available, a reasonable approach is to discuss the risks and benefits of various treatment strategies with patients, and engage a multidisciplinary team (cardiologist, nephrologist, primary care provider, pharmacist) in the decision making process.
 

 

Additional reading

January CT et al. 2019 AHA/ACC/HRS focused update of the 2014 AHA/ACC/HRS guideline for the management of patients with atrial fibrillation: A report of the American College of Cardiology/American Heart Association Task Force on Clinical Practice Guidelines and the Heart Rhythm Society. Circulation. 2019;139. doi: 1161/CIR.0000000000000665.

Nigwekar SU et al. Long-term anticoagulation for patient receiving dialysis: Tilting the benefit to risk ratio? Circulation. 2018 Oct 9;138(15):1530-3. doi: 10.1161/CIRCULATIONAHA.118.037091.

Garlo KG et al. Demystifying the benefits and harms of anticoagulation for atrial fibrillation in chronic kidney disease. Clin J Am Soc Nephrol 2019;14:125-36. doi: 10.2215/CJN.06430518.

Quiz

Two days ago you admitted a 72-year-old woman with end-stage renal disease on dialysis who had developed new-onset atrial fibrillation causing a mild acute diastolic congestive heart failure exacerbation. Transthoracic ECG showed a preserved left ventricular ejection fraction and no significant valvular disease. After two sessions of dialysis in the hospital and initiation of a beta-blocker for control of her heart rate, she is stable and ready for discharge. Her discharge weight is 75 kg.

Which of the following recommendations should you make to this patient regarding anticoagulation for prevention of stroke and systemic embolism from atrial fibrillation?

A. Take warfarin with a international normalized ratio goal of 2.5.

B. Take apixaban 2.5 mg twice a day.

C. Take apixaban 5 mg twice a day.

D. Discuss the risks/benefits of various treatment approaches with the patient, and involve the hospital pharmacist as well as the patient’s nephrologist, cardiologist, and/or primary care provider in the decision making process to reach a consensus and to ensure a safe follow-up plan.

The best answer is D. While A, B, and C are all reasonable approaches based on the available data and current guidelines, the best approach is to involve the patient and the multidisciplinary team in the decision making process. When more clinical trial data becomes available in the future, the optimal approach to managing patients such as this one may become clearer, but until then it makes sense to take into account individual patient characteristics and patient preferences.

 

Case

A 78-year-old woman with end-stage renal disease (ESRD) is hospitalized with cellulitis and is incidentally found to be in atrial fibrillation. She does not have a history of mitral stenosis, nor does she have a prosthetic valve. She does have a history of hypertension, diabetes, and prior stroke without residual deficits.

Dr. Faye Farber
Dr. Faye Farber

After counseling her about the risk of stroke associated with atrial fibrillation (AFib) she makes it clear she is interested in pharmacologic therapy to minimize her risk of stroke and asks what medication you would recommend for anticoagulation.

Brief overview of the issue

Anticoagulation for AFib is indicated for stroke prophylaxis in patients with an elevated risk of stroke. The CHA2DS2-VASc score is useful in calculating an individual patient’s risk of stroke and as a decision tool to determine who would benefit from anticoagulation, and it is recommended in the American Heart Association guidelines.1

Low-risk patients (CHA2DS2-VASc score of 0 in men or 1 in women) should not be started on anticoagulation for stroke prophylaxis. For anyone with a risk factor, other than being female, anticoagulation is indicated and should be considered.

Dr. Neil Stafford of the Division of Hospital Medicine, Duke University Health System, Durham, NC
Dr. Neil Stafford

The guideline recommends anticoagulant therapy, not antiplatelet agents. For most of the recent past, this has meant a vitamin K antagonist (warfarin) or sometimes a low-molecular-weight heparin injected subcutaneously. Over the past decade, however, with the approval of multiple direct oral anticoagulants (DOACs), nonwarfarin oral anticoagulation has grown in popularity as the prophylactic medication of choice.2

While the data for patients with preserved renal function is robust, there is far less data to guide decision making for patients with end-stage renal disease.

Overview of the data

Until the introduction of DOACs, warfarin was the main agent used for stroke prophylaxis in patients with end-stage kidney disease and AFib. Professional guidelines favored warfarin for these patients who were mostly excluded from DOAC trials. Specialized conferences also looked at this issue.

Dr. Suchita Sata

The Kidney Disease: Improving Global Outcomes (KDIGO) Controversies Conference, which reviewed chronic kidney disease and arrhythmias, noted that there were no randomized controlled trials that examined the efficacy and safety of anticoagulation in chronic kidney disease patients with estimated creatinine clearance less than 30 mL/min. They remarked that there was insufficient high-quality evidence to recommend warfarin for the prevention of stroke in patients with AFib and dialysis-dependent chronic kidney disease.

Since, according to other trials, DOACs had better safety profiles in other populations, the conference noted that lower-dose apixaban (2.5 mg orally twice daily) or rivaroxaban (15 mg daily) may be considered in this population until clinical safety data were available. Furthermore, the conference recommended that these patients be treated with a multidisciplinary approach in regards to anticoagulation and have an annual reevaluation of treatment goals, along with a risk-benefit assessment.3

Dr. Rami Abdo of Division of Hospital Medicine, Duke University Health System, Durham, NC
Dr. Rami Abdo

Since the publication of the 2018 AHA guidelines and the guidance document that resulted from the KDIGO conference, additional research has been published comparing anticoagulation with a DOAC versus warfarin for AFib in patients with ESRD.

“Outcomes associated with apixaban use in patients with end-stage kidney disease and atrial fibrillation in the United States” was an observational, retrospective, cohort study that compared outcomes in dialysis patients who took warfarin for AFib with those who took apixaban.4 Patients’ data was taken from the U.S. Renal Data System database and were included in the final analysis if they had ESRD, a recent diagnosis of AFib or atrial flutter, and a new prescription for either warfarin or apixaban. Outcome measures were stroke or systemic embolism, major bleeding (critical site, transfusion, or death), gastrointestinal bleeding, intracranial bleeding, or death. Drug usage and compliance were assessed using Medicare Part D prescription information.

Dr. Megan Brooks, assistant professor of medicine, Duke University, Durham, N.C.
Dr. Megan Brooks

A total of 25,523 patients met the inclusion/exclusion criteria and had taken either warfarin (n = 23,172) or apixaban (n = 2,351). To account for selection bias in these cohorts, a subset of the warfarin patients was selected based on prognostic score matching. The prognostic score was calculated from the baseline characteristics (which included age, stroke history, diabetes, smoking, antiplatelet medication, liver disease, prior bleeding, and CHA2DS2-VASc score). Kaplan-Meier and Cox regression analysis were used to give hazard ratios and 95% confidence intervals for each outcome measure. Prespecified subgroup analyses were conducted to compare apixaban doses, where 44% were prescribed 5 mg b.i.d. and 56% were prescribed 2.5 mg b.i.d..

In the study, patients in the apixaban group had a significantly lower risk of major bleeding as compared with the warfarin group (HR, 0.72; 95% CI, 0.59-0.87; P less than .001) with overall high rates of major bleeding in both groups at 19.7 and 22.9 per 100 patient-years in the apixaban group and warfarin group, respectively. There was no difference in the rate of stroke/systemic embolism between patients receiving apixaban and warfarin (HR, 0.88; 95% CI, 0.69-1.12; P = .29). There was a nonsignificant trend toward decreased risk of GI bleeding in the apixaban group and no significant differences between the groups in the rates of intracranial bleeding. Apixaban was also associated with a nonsignificant trend toward lower risk of mortality (HR, 0.85; 95% CI, 0.71-1.01; P = .06).

Dr. Shree Menon of the Division of Hospital Medicine, Duke University Health System, Durham, NC
Dr. Shree Menon

Notably, censoring rates because of expired prescriptions or a 1-month gap between prescriptions were high in both groups and the majority of censoring occurred within the first 12 months. Additionally, in dose specific analyses, patients receiving the 5-mg, twice-daily dose were found to have statistically significant decreases in risk of stroke/systemic embolism (P = .035) and mortality (P = .005) as compared with the 2.5-mg, twice-daily dose without significant differences in GI or intracranial bleeding.

There are three ongoing, open-label, randomized, controlled trials examining anticoagulation for nonvalvular AFib in patients with ESRD on hemodialysis with two comparing apixaban to warfarin (or derivative) and the other warfarin versus no anticoagulation.5 All trials are in adult patients with documented AFib and CHA2DS2-VASc score of at least 2. AKADIA (Germany based) plans to enroll 222 patients and compares a vitamin K antagonist (INR goal, 2-3) with 2.5-mg b.i.d. apixaban patients with ESRD on hemodialysis for at least 3 months with primary outcome of major and clinically relevant nonmajor bleeding and secondary outcome of thromboembolic events, as well as apixaban levels pre- and post hemodialysis.

Dr. Adam Wachter
Dr. Adam Wachter

RENAL-AF (U.S. based) plans to enrolled 762 patients and compares 5-mg b.i.d. apixaban (with 2.5 mg for selected patients) with warfarin in people of chronic hemodialysis with primary outcome of days to first major or clinically relevant nonmajor bleeding event and secondary outcome of stroke, systemic embolism, mortality, adherence and plasma apixaban levels. AVKDIAL (France based) plans to enroll 855 patients and compares no anticoagulation with vitamin K antagonists in patients on hemodialysis for at least 1 month, with primary outcome of cumulative incidence of severe bleeding and thrombosis.

 

 

Application of the data to our original case

Our patient is Medicare age with ESRD and newly diagnosed nonvalvular AFib. Recent data suggests apixaban could be used for stroke prevention instead of the prior standard of care, warfarin. This approach is supported in the 2019 guidelines.1

Dr. Poonam Sharma of Duke University Health System, Durham, N.C.
Dr. Poonam Sharma

Patients with ESRD have an increased risk of bleeding and apixaban was shown to have less bleeding complications than warfarin in this analysis. However, only standard-dose apixaban was associated with a statistically significant lower risk of stroke/systemic embolism, major bleeding, and death. Reduced-dose apixaban had a lower risk of major bleeding but no difference for stroke/systemic embolism or death. Reduced-dose apixaban is used for patients who have two out of the following three criteria: aged at least 80 years, weight of at least 60 kg, and creatinine of at least 1.5 mg/dL. Therefore, many Medicare-age patients with ESRD would not be indicated for the dose of apixaban that was shown to improve the most important outcomes of stroke/SE and death.

It may still be beneficial to use apixaban in this patient since it appears to work as well as warfarin for stroke/systemic embolism prevention with less bleeding complications.

Bottom line

For patients who have decided to pursue an anticoagulation strategy for stroke prevention in AFib and have end-stage renal disease, either warfarin or apixaban are sensible options.

Dr. Farber is a medical instructor at Duke University Health System in Durham, N.C. Dr. Stafford is a medical instructor at Duke University. Dr. Sata is assistant professor of medicine at Duke University. Dr. Abdo and Dr. Menon are hospitalists at Duke University. Dr. Brooks is assistant professor of medicine at Duke University. Dr. Wachter is associate medical director at Duke Regional Hospital and assistant professor of medicine at Duke University. Dr. Sharma is associate medical director for clinical education in hospital medicine at Duke Regional Hospital and assistant professor of medicine at Duke University.

References

1. January CT et al. 2019 AHA/ACC/HRS focused update of the 2014 AHA/ACC/HRS guideline for the management of patients with atrial fibrillation: A report of the American College of Cardiology/American Heart Association Task Force on Clinical Practice Guidelines and the Heart Rhythm Society. Circulation. 2019;139. doi: 1161/CIR.0000000000000665.

2. Lippi G et al. Direct oral anticoagulants: Analysis of worldwide use and popularity using Google Trends. Ann Transl Med. 2017 Aug; 5(16):322. doi: 10.21037/atm.2017.06.65.

3. Turakhia MP et al. Chronic kidney disease and arrhythmias: Conclusions from a Kidney Disease: Improving Global Outcomes (KDIGO) Controversies Conference. Eur Heart J. 2018 Jun 21;39(24):2314-25. doi: 10.1093/eurheartj/ehy060.

4. Siontis KC et al. Outcomes associated with apixaban use in patients with end-stage kidney disease and atrial fibrillation in the United States. Circulation. 2018 Oct 9;138(15):1519-29. doi: 10.1161/CIRCULATIONAHA.118.035418.

5. Nigwekar SU et al. Long-term anticoagulation for patient receiving dialysis: Tilting the benefit-to-risk ratio? Circulation. 2018 Oct 9;138(15):1530-3. doi: 10.1161/CIRCULATIONAHA.118.037091.

Key points

  • According to 2019 American Heart Association guidelines, warfarin or apixaban are reasonable options for stroke prevention for patients who have end-stage renal disease and who plan for anticoagulation because of atrial fibrillation.
  • Recent observational data suggests that apixaban may be safer than warfarin in this population.
  • Several randomized, controlled trials are ongoing that may help determine the optimal agent to use in this setting.
  • Until more definitive data is available, a reasonable approach is to discuss the risks and benefits of various treatment strategies with patients, and engage a multidisciplinary team (cardiologist, nephrologist, primary care provider, pharmacist) in the decision making process.
 

 

Additional reading

January CT et al. 2019 AHA/ACC/HRS focused update of the 2014 AHA/ACC/HRS guideline for the management of patients with atrial fibrillation: A report of the American College of Cardiology/American Heart Association Task Force on Clinical Practice Guidelines and the Heart Rhythm Society. Circulation. 2019;139. doi: 1161/CIR.0000000000000665.

Nigwekar SU et al. Long-term anticoagulation for patient receiving dialysis: Tilting the benefit to risk ratio? Circulation. 2018 Oct 9;138(15):1530-3. doi: 10.1161/CIRCULATIONAHA.118.037091.

Garlo KG et al. Demystifying the benefits and harms of anticoagulation for atrial fibrillation in chronic kidney disease. Clin J Am Soc Nephrol 2019;14:125-36. doi: 10.2215/CJN.06430518.

Quiz

Two days ago you admitted a 72-year-old woman with end-stage renal disease on dialysis who had developed new-onset atrial fibrillation causing a mild acute diastolic congestive heart failure exacerbation. Transthoracic ECG showed a preserved left ventricular ejection fraction and no significant valvular disease. After two sessions of dialysis in the hospital and initiation of a beta-blocker for control of her heart rate, she is stable and ready for discharge. Her discharge weight is 75 kg.

Which of the following recommendations should you make to this patient regarding anticoagulation for prevention of stroke and systemic embolism from atrial fibrillation?

A. Take warfarin with a international normalized ratio goal of 2.5.

B. Take apixaban 2.5 mg twice a day.

C. Take apixaban 5 mg twice a day.

D. Discuss the risks/benefits of various treatment approaches with the patient, and involve the hospital pharmacist as well as the patient’s nephrologist, cardiologist, and/or primary care provider in the decision making process to reach a consensus and to ensure a safe follow-up plan.

The best answer is D. While A, B, and C are all reasonable approaches based on the available data and current guidelines, the best approach is to involve the patient and the multidisciplinary team in the decision making process. When more clinical trial data becomes available in the future, the optimal approach to managing patients such as this one may become clearer, but until then it makes sense to take into account individual patient characteristics and patient preferences.

Publications
Publications
Topics
Article Type
Sections
Disallow All Ads
Content Gating
No Gating (article Unlocked/Free)
Alternative CME
Disqus Comments
Default
Use ProPublica
Hide sidebar & use full width
render the right sidebar.

Metformin Continues to Be First-Line Therapy for Type 2 Diabetes

Article Type
Changed
Thu, 12/15/2022 - 16:01
Display Headline
Metformin Continues to Be First-Line Therapy for Type 2 Diabetes

Clinical question: Which medications are most safe and effective at managing type 2 diabetes?

Background: Patients and practitioners need an updated review of the evidence to select the optimal medication for type 2 diabetes management.

Study design: Systematic review.

Synopsis: The authors reviewed 179 trials and 25 observational studies. When comparing metformin to sulfonylureas, metformin was associated with less cardiovascular mortality.

However, when trying to make comparisons based on all-cause mortality or microvascular complications, the evidence is limited. Improvements in hemoglobin A1c levels are similar when comparing different monotherapy options, and low blood sugar was most common with sulfonylureas. The short duration of many trials limits the ability to provide better data on long-term outcomes.

Bottom line: Metformin remains the first-line agent for type 2 diabetes management.

Citation: Maruthur NM, Tseng E, Hutfless S, et al. Diabetes medications as monotherapy or metformin-based combination therapy for type 2 diabetes: a systemic review and meta-analysis. Ann Intern Med. 2016;164(1):740-751.

Short Take

Patients Discharge Readiness May Not Be Adequately Assessed and/or Addressed During Hospitalization

Prospective observational study found unresolved barriers to discharge were common in at least 90% of patients. Patients frequently cited issues including unresolved pain, lack of understanding around discharge plans, and ability to provide self-care.

Citation: Harrison JD, Greysen RS, Jacolbia R, Nguyen A, Auerbach AD. Not ready, not set…discharge: patient-reported barriers to discharge readiness at an academic medical center [published online ahead of print April 15, 2016]. J Hosp Med. doi:10.1002/jhm.2591.

Issue
The Hospitalist - 2016(07)
Publications
Sections

Clinical question: Which medications are most safe and effective at managing type 2 diabetes?

Background: Patients and practitioners need an updated review of the evidence to select the optimal medication for type 2 diabetes management.

Study design: Systematic review.

Synopsis: The authors reviewed 179 trials and 25 observational studies. When comparing metformin to sulfonylureas, metformin was associated with less cardiovascular mortality.

However, when trying to make comparisons based on all-cause mortality or microvascular complications, the evidence is limited. Improvements in hemoglobin A1c levels are similar when comparing different monotherapy options, and low blood sugar was most common with sulfonylureas. The short duration of many trials limits the ability to provide better data on long-term outcomes.

Bottom line: Metformin remains the first-line agent for type 2 diabetes management.

Citation: Maruthur NM, Tseng E, Hutfless S, et al. Diabetes medications as monotherapy or metformin-based combination therapy for type 2 diabetes: a systemic review and meta-analysis. Ann Intern Med. 2016;164(1):740-751.

Short Take

Patients Discharge Readiness May Not Be Adequately Assessed and/or Addressed During Hospitalization

Prospective observational study found unresolved barriers to discharge were common in at least 90% of patients. Patients frequently cited issues including unresolved pain, lack of understanding around discharge plans, and ability to provide self-care.

Citation: Harrison JD, Greysen RS, Jacolbia R, Nguyen A, Auerbach AD. Not ready, not set…discharge: patient-reported barriers to discharge readiness at an academic medical center [published online ahead of print April 15, 2016]. J Hosp Med. doi:10.1002/jhm.2591.

Clinical question: Which medications are most safe and effective at managing type 2 diabetes?

Background: Patients and practitioners need an updated review of the evidence to select the optimal medication for type 2 diabetes management.

Study design: Systematic review.

Synopsis: The authors reviewed 179 trials and 25 observational studies. When comparing metformin to sulfonylureas, metformin was associated with less cardiovascular mortality.

However, when trying to make comparisons based on all-cause mortality or microvascular complications, the evidence is limited. Improvements in hemoglobin A1c levels are similar when comparing different monotherapy options, and low blood sugar was most common with sulfonylureas. The short duration of many trials limits the ability to provide better data on long-term outcomes.

Bottom line: Metformin remains the first-line agent for type 2 diabetes management.

Citation: Maruthur NM, Tseng E, Hutfless S, et al. Diabetes medications as monotherapy or metformin-based combination therapy for type 2 diabetes: a systemic review and meta-analysis. Ann Intern Med. 2016;164(1):740-751.

Short Take

Patients Discharge Readiness May Not Be Adequately Assessed and/or Addressed During Hospitalization

Prospective observational study found unresolved barriers to discharge were common in at least 90% of patients. Patients frequently cited issues including unresolved pain, lack of understanding around discharge plans, and ability to provide self-care.

Citation: Harrison JD, Greysen RS, Jacolbia R, Nguyen A, Auerbach AD. Not ready, not set…discharge: patient-reported barriers to discharge readiness at an academic medical center [published online ahead of print April 15, 2016]. J Hosp Med. doi:10.1002/jhm.2591.

Issue
The Hospitalist - 2016(07)
Issue
The Hospitalist - 2016(07)
Publications
Publications
Article Type
Display Headline
Metformin Continues to Be First-Line Therapy for Type 2 Diabetes
Display Headline
Metformin Continues to Be First-Line Therapy for Type 2 Diabetes
Sections
Disallow All Ads
Content Gating
No Gating (article Unlocked/Free)

Reevaluating Cardiovascular Risk after TIA

Article Type
Changed
Thu, 12/15/2022 - 16:01
Display Headline
Reevaluating Cardiovascular Risk after TIA

Clinical question: What is the prognosis of patients who have a TIA or minor stroke?

Background: Prior studies had estimated the risk in the three months following a TIA or minor stroke of having a stroke or acute coronary syndrome (ACS) as 12% to 20%, but this may not reflect the risk of modern patients receiving the current standards of care.

Study design: Prospective observational registry of patients with recent TIA or minor stroke.

Setting: International, including 21 countries.

Synopsis: Adults with recent TIA or minor stroke were included in this multi-center, international registry, and one-year outcomes were reported. At one year, the Kaplan-Meier estimated event rate for the combined outcome of stroke, ACS, or death from cardiovascular causes was 6.2%. The risk of the cardiovascular events was found to be lower than previously reported, suggesting an improvement in outcomes with current interventions. Elevated ABCD2 score, infarction seen on brain imaging, and large-artery atherosclerosis were each associated with higher risk.

Bottom line: Elevated ABCD2 score, brain imaging findings, and large-artery atherosclerosis suggest increased risk for recurrent stroke.

Citation: Amarenco P, Lavallée PC, Labreuche J, et al. One-year risk of stroke after transient ischemic attack or minor stroke. N Engl J Med. 2016;374(16):1533-1542.

Issue
The Hospitalist - 2016(07)
Publications
Sections

Clinical question: What is the prognosis of patients who have a TIA or minor stroke?

Background: Prior studies had estimated the risk in the three months following a TIA or minor stroke of having a stroke or acute coronary syndrome (ACS) as 12% to 20%, but this may not reflect the risk of modern patients receiving the current standards of care.

Study design: Prospective observational registry of patients with recent TIA or minor stroke.

Setting: International, including 21 countries.

Synopsis: Adults with recent TIA or minor stroke were included in this multi-center, international registry, and one-year outcomes were reported. At one year, the Kaplan-Meier estimated event rate for the combined outcome of stroke, ACS, or death from cardiovascular causes was 6.2%. The risk of the cardiovascular events was found to be lower than previously reported, suggesting an improvement in outcomes with current interventions. Elevated ABCD2 score, infarction seen on brain imaging, and large-artery atherosclerosis were each associated with higher risk.

Bottom line: Elevated ABCD2 score, brain imaging findings, and large-artery atherosclerosis suggest increased risk for recurrent stroke.

Citation: Amarenco P, Lavallée PC, Labreuche J, et al. One-year risk of stroke after transient ischemic attack or minor stroke. N Engl J Med. 2016;374(16):1533-1542.

Clinical question: What is the prognosis of patients who have a TIA or minor stroke?

Background: Prior studies had estimated the risk in the three months following a TIA or minor stroke of having a stroke or acute coronary syndrome (ACS) as 12% to 20%, but this may not reflect the risk of modern patients receiving the current standards of care.

Study design: Prospective observational registry of patients with recent TIA or minor stroke.

Setting: International, including 21 countries.

Synopsis: Adults with recent TIA or minor stroke were included in this multi-center, international registry, and one-year outcomes were reported. At one year, the Kaplan-Meier estimated event rate for the combined outcome of stroke, ACS, or death from cardiovascular causes was 6.2%. The risk of the cardiovascular events was found to be lower than previously reported, suggesting an improvement in outcomes with current interventions. Elevated ABCD2 score, infarction seen on brain imaging, and large-artery atherosclerosis were each associated with higher risk.

Bottom line: Elevated ABCD2 score, brain imaging findings, and large-artery atherosclerosis suggest increased risk for recurrent stroke.

Citation: Amarenco P, Lavallée PC, Labreuche J, et al. One-year risk of stroke after transient ischemic attack or minor stroke. N Engl J Med. 2016;374(16):1533-1542.

Issue
The Hospitalist - 2016(07)
Issue
The Hospitalist - 2016(07)
Publications
Publications
Article Type
Display Headline
Reevaluating Cardiovascular Risk after TIA
Display Headline
Reevaluating Cardiovascular Risk after TIA
Sections
Disallow All Ads
Content Gating
No Gating (article Unlocked/Free)

Single Dose of Dexamethasone Not an Alternative to ‘Steroid Burst’ for Acute Asthma Treatment

Article Type
Changed
Thu, 12/15/2022 - 16:01
Display Headline
Single Dose of Dexamethasone Not an Alternative to ‘Steroid Burst’ for Acute Asthma Treatment

Clinical question: Is one dose of dexamethasone comparable to five days of prednisone for treating mild-to-moderate asthma exacerbations?

Background: Corticosteroids are the mainstay of initial treatment for asthma exacerbations. The National Heart, Lung, and Blood Institute recommends a minimum of five days of prednisone, though studies have shown incomplete adherence to prolonged therapies. Dexamethasone has a longer duration of action than prednisone.

Study design: Randomized, controlled, double-blinded trial.

Setting: Urban, safety-net, teaching hospital.

Synopsis: The study included 376 adults ages 18–55 presenting to the emergency department for a mild-to-moderate asthma exacerbation who were randomized to two treatment courses of corticosteroids: one 12 mg dose of oral dexamethasone followed by four days of placebo versus five days of 60 mg of oral prednisone. Two weeks later, a telephone survey asked if they had relapsed and had to seek medical attention. This study did not show noninferiority of the dexamethasone option compared to the standard of care. Specifically, it showed a 12.1% relapse rate in the dexamethasone group versus a 9.8% relapse rate for prednisone (95% CI, -4.1% to 8.6%).

This was a small study looking at adults without other chronic lung diseases or diabetes. The authors did not include those patients who were either lost to follow-up (20% of those initially randomized) or ultimately admitted after their emergency department course.

Hospitalists who care for patients with asthma should look to the current standards of corticosteroid selection and duration to minimize clinical relapses and possibly readmissions.

Bottom line: One large dose of dexamethasone is inferior to the standard five days of prednisone for treating acute asthma exacerbations in adults.

Citation: Rehrer MW, Liu B, Rodriguez M, Lam J, Alter HJ. A randomized controlled noninferiority trial of single dose of oral dexamethasone versus 5 days of oral prednisone in acute adult asthma [published online ahead of print April 14, 2016]. Ann Emerg Med. doi:10.1016/j.annemergmed.2016.03.017.

Short Take

Guideline Recommends ED Asthma Management Associated with Shorter Inpatient Stay

Observational study found ED treatment concordance with four guideline-based processes for acute asthma treatment (inhaled beta-agonists, inhaled anticholinergics, systemic corticosteroids, and avoidance of methylxanthines) is associated with a 17% shorter hospital length of stay.

Citation: Hasegawa K, Brenner BE, Nowak RM, et al. Association of guideline-concordant acute asthma care in the emergency department with shorter hospital length of stay: a multicenter observational study. Acad Emerg Med. 2016;23(5):616-622.

Issue
The Hospitalist - 2016(06)
Publications
Topics
Sections

Clinical question: Is one dose of dexamethasone comparable to five days of prednisone for treating mild-to-moderate asthma exacerbations?

Background: Corticosteroids are the mainstay of initial treatment for asthma exacerbations. The National Heart, Lung, and Blood Institute recommends a minimum of five days of prednisone, though studies have shown incomplete adherence to prolonged therapies. Dexamethasone has a longer duration of action than prednisone.

Study design: Randomized, controlled, double-blinded trial.

Setting: Urban, safety-net, teaching hospital.

Synopsis: The study included 376 adults ages 18–55 presenting to the emergency department for a mild-to-moderate asthma exacerbation who were randomized to two treatment courses of corticosteroids: one 12 mg dose of oral dexamethasone followed by four days of placebo versus five days of 60 mg of oral prednisone. Two weeks later, a telephone survey asked if they had relapsed and had to seek medical attention. This study did not show noninferiority of the dexamethasone option compared to the standard of care. Specifically, it showed a 12.1% relapse rate in the dexamethasone group versus a 9.8% relapse rate for prednisone (95% CI, -4.1% to 8.6%).

This was a small study looking at adults without other chronic lung diseases or diabetes. The authors did not include those patients who were either lost to follow-up (20% of those initially randomized) or ultimately admitted after their emergency department course.

Hospitalists who care for patients with asthma should look to the current standards of corticosteroid selection and duration to minimize clinical relapses and possibly readmissions.

Bottom line: One large dose of dexamethasone is inferior to the standard five days of prednisone for treating acute asthma exacerbations in adults.

Citation: Rehrer MW, Liu B, Rodriguez M, Lam J, Alter HJ. A randomized controlled noninferiority trial of single dose of oral dexamethasone versus 5 days of oral prednisone in acute adult asthma [published online ahead of print April 14, 2016]. Ann Emerg Med. doi:10.1016/j.annemergmed.2016.03.017.

Short Take

Guideline Recommends ED Asthma Management Associated with Shorter Inpatient Stay

Observational study found ED treatment concordance with four guideline-based processes for acute asthma treatment (inhaled beta-agonists, inhaled anticholinergics, systemic corticosteroids, and avoidance of methylxanthines) is associated with a 17% shorter hospital length of stay.

Citation: Hasegawa K, Brenner BE, Nowak RM, et al. Association of guideline-concordant acute asthma care in the emergency department with shorter hospital length of stay: a multicenter observational study. Acad Emerg Med. 2016;23(5):616-622.

Clinical question: Is one dose of dexamethasone comparable to five days of prednisone for treating mild-to-moderate asthma exacerbations?

Background: Corticosteroids are the mainstay of initial treatment for asthma exacerbations. The National Heart, Lung, and Blood Institute recommends a minimum of five days of prednisone, though studies have shown incomplete adherence to prolonged therapies. Dexamethasone has a longer duration of action than prednisone.

Study design: Randomized, controlled, double-blinded trial.

Setting: Urban, safety-net, teaching hospital.

Synopsis: The study included 376 adults ages 18–55 presenting to the emergency department for a mild-to-moderate asthma exacerbation who were randomized to two treatment courses of corticosteroids: one 12 mg dose of oral dexamethasone followed by four days of placebo versus five days of 60 mg of oral prednisone. Two weeks later, a telephone survey asked if they had relapsed and had to seek medical attention. This study did not show noninferiority of the dexamethasone option compared to the standard of care. Specifically, it showed a 12.1% relapse rate in the dexamethasone group versus a 9.8% relapse rate for prednisone (95% CI, -4.1% to 8.6%).

This was a small study looking at adults without other chronic lung diseases or diabetes. The authors did not include those patients who were either lost to follow-up (20% of those initially randomized) or ultimately admitted after their emergency department course.

Hospitalists who care for patients with asthma should look to the current standards of corticosteroid selection and duration to minimize clinical relapses and possibly readmissions.

Bottom line: One large dose of dexamethasone is inferior to the standard five days of prednisone for treating acute asthma exacerbations in adults.

Citation: Rehrer MW, Liu B, Rodriguez M, Lam J, Alter HJ. A randomized controlled noninferiority trial of single dose of oral dexamethasone versus 5 days of oral prednisone in acute adult asthma [published online ahead of print April 14, 2016]. Ann Emerg Med. doi:10.1016/j.annemergmed.2016.03.017.

Short Take

Guideline Recommends ED Asthma Management Associated with Shorter Inpatient Stay

Observational study found ED treatment concordance with four guideline-based processes for acute asthma treatment (inhaled beta-agonists, inhaled anticholinergics, systemic corticosteroids, and avoidance of methylxanthines) is associated with a 17% shorter hospital length of stay.

Citation: Hasegawa K, Brenner BE, Nowak RM, et al. Association of guideline-concordant acute asthma care in the emergency department with shorter hospital length of stay: a multicenter observational study. Acad Emerg Med. 2016;23(5):616-622.

Issue
The Hospitalist - 2016(06)
Issue
The Hospitalist - 2016(06)
Publications
Publications
Topics
Article Type
Display Headline
Single Dose of Dexamethasone Not an Alternative to ‘Steroid Burst’ for Acute Asthma Treatment
Display Headline
Single Dose of Dexamethasone Not an Alternative to ‘Steroid Burst’ for Acute Asthma Treatment
Sections
Disallow All Ads
Content Gating
No Gating (article Unlocked/Free)

Interhospital Transfer Handoff Practice Variance at U.S. Tertiary Care Centers

Article Type
Changed
Thu, 12/15/2022 - 16:01
Display Headline
Interhospital Transfer Handoff Practice Variance at U.S. Tertiary Care Centers

Clinical question: How do interhospital handoff practices differ among U.S. tertiary care centers, and what challenges and innovations have providers encountered?

Background: Little has been studied regarding interhospital transfers. Many centers differ in the processes they follow, and well-delineated national guidelines are lacking. Adverse events occur in up to 30% of transfers. Standardization of these handoffs has been shown to reduce preventable errors and near misses.

Study design: Survey of convenience sample of institutions.

Setting: Transfer center directors from 32 tertiary care centers in the U.S.

Synopsis: The authors surveyed directors of 32 transfer centers between 2013 and 2015. Hospitals were selected from a nationally ranked list as well as those comparable to the authors’ own institutions. The median number of patients transferred per month was 700.

Only 23% of hospitals surveyed identified significant EHR interoperability. Almost all required three-way recorded discussion between transfer center staff and referring and accepting physicians. Only 29% had available objective clinical information to share. Only 23% recorded a three-way nursing handoff, and only 32% used their EHR to document the transfer process and share clinical information among providers.

Innovations included electronic transfer notes, a standardized system of feedback to referring hospitals, automatic internal review for adverse events and delayed transfers, and use of a scorecard with key measures shared with stakeholders.

Barriers noted included complexity, acuity, and lack of continuity. Increased use of EHRs, checklists, and common processes were identified as best practices.

Limitations of the study included reliance on verbal qualitative data, a single investigator doing most of the discussions, and possible sampling bias.

Bottom line: Interhospital transfer practices at academic tertiary care centers vary widely, and optimizing and aligning practices between sending and receiving hospitals may improve efficiency and patient outcomes.

References: Herrigel DJ, Carroll M, Fanning C, Steinberg MB, Parikh A, Usher M. Interhospital transfer handoff practices among US tertiary care centers: a descriptive survey. J Hosp Med. 2016;11(6):413-417.

Issue
The Hospitalist - 2016(06)
Publications
Sections

Clinical question: How do interhospital handoff practices differ among U.S. tertiary care centers, and what challenges and innovations have providers encountered?

Background: Little has been studied regarding interhospital transfers. Many centers differ in the processes they follow, and well-delineated national guidelines are lacking. Adverse events occur in up to 30% of transfers. Standardization of these handoffs has been shown to reduce preventable errors and near misses.

Study design: Survey of convenience sample of institutions.

Setting: Transfer center directors from 32 tertiary care centers in the U.S.

Synopsis: The authors surveyed directors of 32 transfer centers between 2013 and 2015. Hospitals were selected from a nationally ranked list as well as those comparable to the authors’ own institutions. The median number of patients transferred per month was 700.

Only 23% of hospitals surveyed identified significant EHR interoperability. Almost all required three-way recorded discussion between transfer center staff and referring and accepting physicians. Only 29% had available objective clinical information to share. Only 23% recorded a three-way nursing handoff, and only 32% used their EHR to document the transfer process and share clinical information among providers.

Innovations included electronic transfer notes, a standardized system of feedback to referring hospitals, automatic internal review for adverse events and delayed transfers, and use of a scorecard with key measures shared with stakeholders.

Barriers noted included complexity, acuity, and lack of continuity. Increased use of EHRs, checklists, and common processes were identified as best practices.

Limitations of the study included reliance on verbal qualitative data, a single investigator doing most of the discussions, and possible sampling bias.

Bottom line: Interhospital transfer practices at academic tertiary care centers vary widely, and optimizing and aligning practices between sending and receiving hospitals may improve efficiency and patient outcomes.

References: Herrigel DJ, Carroll M, Fanning C, Steinberg MB, Parikh A, Usher M. Interhospital transfer handoff practices among US tertiary care centers: a descriptive survey. J Hosp Med. 2016;11(6):413-417.

Clinical question: How do interhospital handoff practices differ among U.S. tertiary care centers, and what challenges and innovations have providers encountered?

Background: Little has been studied regarding interhospital transfers. Many centers differ in the processes they follow, and well-delineated national guidelines are lacking. Adverse events occur in up to 30% of transfers. Standardization of these handoffs has been shown to reduce preventable errors and near misses.

Study design: Survey of convenience sample of institutions.

Setting: Transfer center directors from 32 tertiary care centers in the U.S.

Synopsis: The authors surveyed directors of 32 transfer centers between 2013 and 2015. Hospitals were selected from a nationally ranked list as well as those comparable to the authors’ own institutions. The median number of patients transferred per month was 700.

Only 23% of hospitals surveyed identified significant EHR interoperability. Almost all required three-way recorded discussion between transfer center staff and referring and accepting physicians. Only 29% had available objective clinical information to share. Only 23% recorded a three-way nursing handoff, and only 32% used their EHR to document the transfer process and share clinical information among providers.

Innovations included electronic transfer notes, a standardized system of feedback to referring hospitals, automatic internal review for adverse events and delayed transfers, and use of a scorecard with key measures shared with stakeholders.

Barriers noted included complexity, acuity, and lack of continuity. Increased use of EHRs, checklists, and common processes were identified as best practices.

Limitations of the study included reliance on verbal qualitative data, a single investigator doing most of the discussions, and possible sampling bias.

Bottom line: Interhospital transfer practices at academic tertiary care centers vary widely, and optimizing and aligning practices between sending and receiving hospitals may improve efficiency and patient outcomes.

References: Herrigel DJ, Carroll M, Fanning C, Steinberg MB, Parikh A, Usher M. Interhospital transfer handoff practices among US tertiary care centers: a descriptive survey. J Hosp Med. 2016;11(6):413-417.

Issue
The Hospitalist - 2016(06)
Issue
The Hospitalist - 2016(06)
Publications
Publications
Article Type
Display Headline
Interhospital Transfer Handoff Practice Variance at U.S. Tertiary Care Centers
Display Headline
Interhospital Transfer Handoff Practice Variance at U.S. Tertiary Care Centers
Sections
Disallow All Ads
Content Gating
No Gating (article Unlocked/Free)

Oral Steroids as Good as NSAIDs for Acute Gout

Article Type
Changed
Thu, 12/15/2022 - 16:01
Display Headline
Oral Steroids as Good as NSAIDs for Acute Gout

Clinical question: Are oral steroids as effective as NSAIDs in treating acute gout?

Background: Two small trials have suggested that oral steroids are as effective as NSAIDs in treating acute gout. Wider acceptance of steroids as first-line agents for acute gout may require more robust evidence supporting their safety and efficacy.

Study design: Multicenter, double-blind, randomized equivalence trial.

Setting: Four EDs in Hong Kong.

Synopsis: The study included 416 patients presenting to the ED with clinically suspected acute gout who were randomized to treatment with either oral indomethacin or oral prednisolone for five days. A research investigator assessed response to therapy in the ED at 30, 60, 90, and 120 minutes after administration of the initial dose of medication. Patients then kept pain-assessment diaries for 14 days after discharge from the ED.

Pain scores were assessed using a visual analog scale of 0 mm (no pain) to 100 mm (worst pain the patient had experienced). Clinically significant changes in pain scores were defined as decreases of >13 mm on the visual analog scale.

The number of patients with clinically significant decreases in pain scores did not differ statistically between groups. Both groups had similar reductions in mean pain scores over the course of the study. Patients in the indomethacin group had a statistically significant increase in minor adverse events. No patients in either group had major adverse events.

Bottom line: Oral prednisolone appears to be a safe and effective first-line agent for the treatment of acute gout.

Citation: Rainer TH, Chen CH, Janssens HJEM, et al. Oral prednisolone in the treatment of acute gout. Ann Intern Med. 2016;164(7):464-471.

Short Take

Rate Control as Effective as Rhythm Control in Postoperative Atrial Fibrillation

This study randomized patients with postoperative atrial fibrillation to rhythm control (using amiodarone and/or direct current cardioversion) or rate control and found neither treatment strategy has a clinical benefit over the other.

Citation: Gillinov AM, Bagiella E, Moskowitz AJ, et al. Rate control versus rhythm control for atrial fibrillation after cardiac surgery. N Engl J Med. 2016;374(20):1911-1921.

Issue
The Hospitalist - 2016(06)
Publications
Topics
Sections

Clinical question: Are oral steroids as effective as NSAIDs in treating acute gout?

Background: Two small trials have suggested that oral steroids are as effective as NSAIDs in treating acute gout. Wider acceptance of steroids as first-line agents for acute gout may require more robust evidence supporting their safety and efficacy.

Study design: Multicenter, double-blind, randomized equivalence trial.

Setting: Four EDs in Hong Kong.

Synopsis: The study included 416 patients presenting to the ED with clinically suspected acute gout who were randomized to treatment with either oral indomethacin or oral prednisolone for five days. A research investigator assessed response to therapy in the ED at 30, 60, 90, and 120 minutes after administration of the initial dose of medication. Patients then kept pain-assessment diaries for 14 days after discharge from the ED.

Pain scores were assessed using a visual analog scale of 0 mm (no pain) to 100 mm (worst pain the patient had experienced). Clinically significant changes in pain scores were defined as decreases of >13 mm on the visual analog scale.

The number of patients with clinically significant decreases in pain scores did not differ statistically between groups. Both groups had similar reductions in mean pain scores over the course of the study. Patients in the indomethacin group had a statistically significant increase in minor adverse events. No patients in either group had major adverse events.

Bottom line: Oral prednisolone appears to be a safe and effective first-line agent for the treatment of acute gout.

Citation: Rainer TH, Chen CH, Janssens HJEM, et al. Oral prednisolone in the treatment of acute gout. Ann Intern Med. 2016;164(7):464-471.

Short Take

Rate Control as Effective as Rhythm Control in Postoperative Atrial Fibrillation

This study randomized patients with postoperative atrial fibrillation to rhythm control (using amiodarone and/or direct current cardioversion) or rate control and found neither treatment strategy has a clinical benefit over the other.

Citation: Gillinov AM, Bagiella E, Moskowitz AJ, et al. Rate control versus rhythm control for atrial fibrillation after cardiac surgery. N Engl J Med. 2016;374(20):1911-1921.

Clinical question: Are oral steroids as effective as NSAIDs in treating acute gout?

Background: Two small trials have suggested that oral steroids are as effective as NSAIDs in treating acute gout. Wider acceptance of steroids as first-line agents for acute gout may require more robust evidence supporting their safety and efficacy.

Study design: Multicenter, double-blind, randomized equivalence trial.

Setting: Four EDs in Hong Kong.

Synopsis: The study included 416 patients presenting to the ED with clinically suspected acute gout who were randomized to treatment with either oral indomethacin or oral prednisolone for five days. A research investigator assessed response to therapy in the ED at 30, 60, 90, and 120 minutes after administration of the initial dose of medication. Patients then kept pain-assessment diaries for 14 days after discharge from the ED.

Pain scores were assessed using a visual analog scale of 0 mm (no pain) to 100 mm (worst pain the patient had experienced). Clinically significant changes in pain scores were defined as decreases of >13 mm on the visual analog scale.

The number of patients with clinically significant decreases in pain scores did not differ statistically between groups. Both groups had similar reductions in mean pain scores over the course of the study. Patients in the indomethacin group had a statistically significant increase in minor adverse events. No patients in either group had major adverse events.

Bottom line: Oral prednisolone appears to be a safe and effective first-line agent for the treatment of acute gout.

Citation: Rainer TH, Chen CH, Janssens HJEM, et al. Oral prednisolone in the treatment of acute gout. Ann Intern Med. 2016;164(7):464-471.

Short Take

Rate Control as Effective as Rhythm Control in Postoperative Atrial Fibrillation

This study randomized patients with postoperative atrial fibrillation to rhythm control (using amiodarone and/or direct current cardioversion) or rate control and found neither treatment strategy has a clinical benefit over the other.

Citation: Gillinov AM, Bagiella E, Moskowitz AJ, et al. Rate control versus rhythm control for atrial fibrillation after cardiac surgery. N Engl J Med. 2016;374(20):1911-1921.

Issue
The Hospitalist - 2016(06)
Issue
The Hospitalist - 2016(06)
Publications
Publications
Topics
Article Type
Display Headline
Oral Steroids as Good as NSAIDs for Acute Gout
Display Headline
Oral Steroids as Good as NSAIDs for Acute Gout
Sections
Disallow All Ads
Content Gating
No Gating (article Unlocked/Free)

Better Reporting Needed to Accurately Estimate Medical Error as Cause of Death in U.S.

Article Type
Changed
Thu, 12/15/2022 - 16:01
Display Headline
Better Reporting Needed to Accurately Estimate Medical Error as Cause of Death in U.S.

Clinical question: What is the contribution of hospital-based medical errors to national mortality in the U.S. compared to other causes listed by the Centers for Disease Control and Prevention (CDC)?

Background: Medical error can contribute to patient mortality. Currently, the annual list of the most common causes of death in the U.S. is compiled by the CDC using the International Classification of Diseases (ICD) codes on death certificates. Deaths caused by errors are unmeasured because medical errors are not included in the death certificate.

Study design: Analysis of existing literature on medical errors.

Setting: U.S. hospitals.

Synopsis: Findings of four studies on U.S. death rates from medical errors published between 2000 and 2008 were synthesized and extrapolated to the total number of U.S. hospital admissions in 2013. This resulted in a mean rate of death from medical errors of 251,454 per year, which is much higher than the annual incidence of 44,000–98,000 deaths published in the 1999 Institute of Medicine report. Comparing these data to the CDC ranking makes medical errors the third-leading cause of death in the U.S.

Although the accuracy of this result is limited to inpatient deaths and as the authors extrapolated the data from other studies, the number is still staggering and highlights the need for systematic measurement of the problem. One simple solution for this could be to have an extra field on the death certificate asking whether a preventable complication stemming from the patient’s medical care contributed to the death.

Bottom line: Medical error as the estimated third-leading cause of the death in the U.S. remains under-recognized, underappreciated, and highly unmeasured.

Citation: Makary MA, Daniel M. Medical error-the third leading cause of death in the US. BMJ. 2016;353:i2139.

Short Take

Isolating C. Difficile Carriers Decreases Hospital-Acquired C. Difficile Infections

In a nonblinded time-series analysis, screening all patients for asymptomatic C. diff carrier status and isolating carriers reduced rates of hospital-acquired C. diff, preventing 62.4% of expected cases.

Citation: Longtin Y, Paquet-Bolduc B, Gilca R, et al. Effect of detecting and isolating Clostridium difficile carriers at hospital admission on the incidence of C difficile infections: a quasi-experimental controlled study. JAMA Inter Med. 2016;176(6):796¬-804.

Issue
The Hospitalist - 2016(06)
Publications
Sections

Clinical question: What is the contribution of hospital-based medical errors to national mortality in the U.S. compared to other causes listed by the Centers for Disease Control and Prevention (CDC)?

Background: Medical error can contribute to patient mortality. Currently, the annual list of the most common causes of death in the U.S. is compiled by the CDC using the International Classification of Diseases (ICD) codes on death certificates. Deaths caused by errors are unmeasured because medical errors are not included in the death certificate.

Study design: Analysis of existing literature on medical errors.

Setting: U.S. hospitals.

Synopsis: Findings of four studies on U.S. death rates from medical errors published between 2000 and 2008 were synthesized and extrapolated to the total number of U.S. hospital admissions in 2013. This resulted in a mean rate of death from medical errors of 251,454 per year, which is much higher than the annual incidence of 44,000–98,000 deaths published in the 1999 Institute of Medicine report. Comparing these data to the CDC ranking makes medical errors the third-leading cause of death in the U.S.

Although the accuracy of this result is limited to inpatient deaths and as the authors extrapolated the data from other studies, the number is still staggering and highlights the need for systematic measurement of the problem. One simple solution for this could be to have an extra field on the death certificate asking whether a preventable complication stemming from the patient’s medical care contributed to the death.

Bottom line: Medical error as the estimated third-leading cause of the death in the U.S. remains under-recognized, underappreciated, and highly unmeasured.

Citation: Makary MA, Daniel M. Medical error-the third leading cause of death in the US. BMJ. 2016;353:i2139.

Short Take

Isolating C. Difficile Carriers Decreases Hospital-Acquired C. Difficile Infections

In a nonblinded time-series analysis, screening all patients for asymptomatic C. diff carrier status and isolating carriers reduced rates of hospital-acquired C. diff, preventing 62.4% of expected cases.

Citation: Longtin Y, Paquet-Bolduc B, Gilca R, et al. Effect of detecting and isolating Clostridium difficile carriers at hospital admission on the incidence of C difficile infections: a quasi-experimental controlled study. JAMA Inter Med. 2016;176(6):796¬-804.

Clinical question: What is the contribution of hospital-based medical errors to national mortality in the U.S. compared to other causes listed by the Centers for Disease Control and Prevention (CDC)?

Background: Medical error can contribute to patient mortality. Currently, the annual list of the most common causes of death in the U.S. is compiled by the CDC using the International Classification of Diseases (ICD) codes on death certificates. Deaths caused by errors are unmeasured because medical errors are not included in the death certificate.

Study design: Analysis of existing literature on medical errors.

Setting: U.S. hospitals.

Synopsis: Findings of four studies on U.S. death rates from medical errors published between 2000 and 2008 were synthesized and extrapolated to the total number of U.S. hospital admissions in 2013. This resulted in a mean rate of death from medical errors of 251,454 per year, which is much higher than the annual incidence of 44,000–98,000 deaths published in the 1999 Institute of Medicine report. Comparing these data to the CDC ranking makes medical errors the third-leading cause of death in the U.S.

Although the accuracy of this result is limited to inpatient deaths and as the authors extrapolated the data from other studies, the number is still staggering and highlights the need for systematic measurement of the problem. One simple solution for this could be to have an extra field on the death certificate asking whether a preventable complication stemming from the patient’s medical care contributed to the death.

Bottom line: Medical error as the estimated third-leading cause of the death in the U.S. remains under-recognized, underappreciated, and highly unmeasured.

Citation: Makary MA, Daniel M. Medical error-the third leading cause of death in the US. BMJ. 2016;353:i2139.

Short Take

Isolating C. Difficile Carriers Decreases Hospital-Acquired C. Difficile Infections

In a nonblinded time-series analysis, screening all patients for asymptomatic C. diff carrier status and isolating carriers reduced rates of hospital-acquired C. diff, preventing 62.4% of expected cases.

Citation: Longtin Y, Paquet-Bolduc B, Gilca R, et al. Effect of detecting and isolating Clostridium difficile carriers at hospital admission on the incidence of C difficile infections: a quasi-experimental controlled study. JAMA Inter Med. 2016;176(6):796¬-804.

Issue
The Hospitalist - 2016(06)
Issue
The Hospitalist - 2016(06)
Publications
Publications
Article Type
Display Headline
Better Reporting Needed to Accurately Estimate Medical Error as Cause of Death in U.S.
Display Headline
Better Reporting Needed to Accurately Estimate Medical Error as Cause of Death in U.S.
Sections
Disallow All Ads
Content Gating
No Gating (article Unlocked/Free)

Hospital Admission, Stroke Clinic Follow-up Improve Outcomes for Patients with Transient Ischemic Attack, Minor Ischemic Stroke

Article Type
Changed
Thu, 12/15/2022 - 16:01
Display Headline
Hospital Admission, Stroke Clinic Follow-up Improve Outcomes for Patients with Transient Ischemic Attack, Minor Ischemic Stroke

Clinical question: How do guideline-based care and outcomes of patients with transient ischemic attack (TIA) and minor ischemic stroke differ among patients admitted to the hospital and discharged from the ED, as well as in those referred versus not referred to stroke prevention clinics following discharge?

Background: Previous research demonstrated that urgent outpatient management strategies for patients with TIA and minor ischemic stroke are superior to standard outpatient care. However, there is less known about how outpatient stroke care compares to inpatient care in terms of outcomes, rapid risk factor identification/modification, and initiation of antithrombotic therapy.

Study design: Retrospective cohort study.

Setting: EDs of acute-care hospitals in Ontario, Canada.

Synopsis: Using the Ontario Stroke Registry, 8,540 patients seen in the ED with TIA or minor ischemic stroke were identified. The use of guideline-based interventions was highest in admitted patients, followed by patients discharged from the ED with stroke clinic follow-up, followed by patients discharged without follow-up. There was no significant difference in one-year mortality between admitted and discharged patients when adjusted for age, sex, and comorbid conditions (adjusted hazard ratio, 1.11; 95% CI, 0.92–1.34). However, stroke clinic referral was associated with a lower risk of one-year mortality compared with those discharged without follow-up (adjusted hazard ratio, 0.49; 95% CI, 0.38–0.64).

Limitations of this study include that it was carried out only in Ontario, where there is a universal healthcare system, which may limit the generalizability of the findings. Additionally, patient information was limited to what was available through the registry, which may mean there were other unmeasurable differences among groups.

Bottom line: Admitted patients with TIA or minor ischemic stroke are more likely to receive guideline-based therapy, and among patients discharged from the ED, referral to stroke clinic improves outcomes, including one-year mortality.

Citation: Kapral MK, Hall R, Fang J, et al. Association between hospitalization and care after transient ischemic attack or minor stroke. Neurology. 2016;86(17):1582-1589.

Issue
The Hospitalist - 2016(06)
Publications
Topics
Sections

Clinical question: How do guideline-based care and outcomes of patients with transient ischemic attack (TIA) and minor ischemic stroke differ among patients admitted to the hospital and discharged from the ED, as well as in those referred versus not referred to stroke prevention clinics following discharge?

Background: Previous research demonstrated that urgent outpatient management strategies for patients with TIA and minor ischemic stroke are superior to standard outpatient care. However, there is less known about how outpatient stroke care compares to inpatient care in terms of outcomes, rapid risk factor identification/modification, and initiation of antithrombotic therapy.

Study design: Retrospective cohort study.

Setting: EDs of acute-care hospitals in Ontario, Canada.

Synopsis: Using the Ontario Stroke Registry, 8,540 patients seen in the ED with TIA or minor ischemic stroke were identified. The use of guideline-based interventions was highest in admitted patients, followed by patients discharged from the ED with stroke clinic follow-up, followed by patients discharged without follow-up. There was no significant difference in one-year mortality between admitted and discharged patients when adjusted for age, sex, and comorbid conditions (adjusted hazard ratio, 1.11; 95% CI, 0.92–1.34). However, stroke clinic referral was associated with a lower risk of one-year mortality compared with those discharged without follow-up (adjusted hazard ratio, 0.49; 95% CI, 0.38–0.64).

Limitations of this study include that it was carried out only in Ontario, where there is a universal healthcare system, which may limit the generalizability of the findings. Additionally, patient information was limited to what was available through the registry, which may mean there were other unmeasurable differences among groups.

Bottom line: Admitted patients with TIA or minor ischemic stroke are more likely to receive guideline-based therapy, and among patients discharged from the ED, referral to stroke clinic improves outcomes, including one-year mortality.

Citation: Kapral MK, Hall R, Fang J, et al. Association between hospitalization and care after transient ischemic attack or minor stroke. Neurology. 2016;86(17):1582-1589.

Clinical question: How do guideline-based care and outcomes of patients with transient ischemic attack (TIA) and minor ischemic stroke differ among patients admitted to the hospital and discharged from the ED, as well as in those referred versus not referred to stroke prevention clinics following discharge?

Background: Previous research demonstrated that urgent outpatient management strategies for patients with TIA and minor ischemic stroke are superior to standard outpatient care. However, there is less known about how outpatient stroke care compares to inpatient care in terms of outcomes, rapid risk factor identification/modification, and initiation of antithrombotic therapy.

Study design: Retrospective cohort study.

Setting: EDs of acute-care hospitals in Ontario, Canada.

Synopsis: Using the Ontario Stroke Registry, 8,540 patients seen in the ED with TIA or minor ischemic stroke were identified. The use of guideline-based interventions was highest in admitted patients, followed by patients discharged from the ED with stroke clinic follow-up, followed by patients discharged without follow-up. There was no significant difference in one-year mortality between admitted and discharged patients when adjusted for age, sex, and comorbid conditions (adjusted hazard ratio, 1.11; 95% CI, 0.92–1.34). However, stroke clinic referral was associated with a lower risk of one-year mortality compared with those discharged without follow-up (adjusted hazard ratio, 0.49; 95% CI, 0.38–0.64).

Limitations of this study include that it was carried out only in Ontario, where there is a universal healthcare system, which may limit the generalizability of the findings. Additionally, patient information was limited to what was available through the registry, which may mean there were other unmeasurable differences among groups.

Bottom line: Admitted patients with TIA or minor ischemic stroke are more likely to receive guideline-based therapy, and among patients discharged from the ED, referral to stroke clinic improves outcomes, including one-year mortality.

Citation: Kapral MK, Hall R, Fang J, et al. Association between hospitalization and care after transient ischemic attack or minor stroke. Neurology. 2016;86(17):1582-1589.

Issue
The Hospitalist - 2016(06)
Issue
The Hospitalist - 2016(06)
Publications
Publications
Topics
Article Type
Display Headline
Hospital Admission, Stroke Clinic Follow-up Improve Outcomes for Patients with Transient Ischemic Attack, Minor Ischemic Stroke
Display Headline
Hospital Admission, Stroke Clinic Follow-up Improve Outcomes for Patients with Transient Ischemic Attack, Minor Ischemic Stroke
Sections
Disallow All Ads
Content Gating
No Gating (article Unlocked/Free)